Vous êtes sur la page 1sur 44

[EDIT] Lecture Topic 1.

02
06/20/2015
ACUTE & CHRONIC INFLAMMATION & TISSUE REPAIR
DR. YABUT

INFLAMMATION
 A nonspecific but predictable response of living tissues or the
entire body to injury
 Not synonymous to infection (a reaction due to pathogens)
 Can be caused by chemical agents, physical forces, living
microbes (pathogens) and many other physiologic and
pathologic (endogenous/exogenous) stimuli that disturb the
normal steady state
 Includes interconnected events
o Thus, it is a dynamic process, evolving through several
phases that last from a few minutes to days or even months
and years.
 Occurs only in multicellular organisms that are capable of
mounting a neurovascular and cellular response to injury
o A coordinated response of the human body involving the
nerves, vessels, blood cells and soluble mediators
 Has a protective role and generally beneficial to the body
o Fever is generally a protective response but if it becomes Figure 1. The inflammatory response to injury. Chemical mediators and cells are
released from plasma following tissue injury. Vasodilation and vascular injury lead to
very high, it can cause death. (If a patient has fever, don’t leakage of fluid into tissues (edema). Platelets are activated to initiate clot formation
treat with paracetamol because the fever is a response of the and hemostasis and to increase vascular permeability via histamine release. Vascular
body to ward off the insulting agent. If it is an infection, treat endothelial cells contribute to clot formation, retract to allow increased vascular
permeability and anchor circulating neutrophils via their adhesion molecules.
with antibiotics instead of antipyretics.) Microbes (red rods) initiate activation of the complement cascade, which, along with
o In pulmonary tuberculosis, there is a protective tissue soluble mediators from macrophages, recruit neutrophils to the site of tissue injury.
reaction but this inflammatory response may erode Neutrophils eliminate microbes and remove damaged tissue so that repair can begin.
pulmonary vessels and cause massive bleeding.
 Occurs only in living tissues
o Necrotic/dead tissues cannot mount an inflammatory
response
o Gangrenous foot: cannot be inflamed and it has to be
amputated
 Forensic science: inflammation is a vital reaction
o If histologic signs of inflammation are found in tissues during
autopsy, this indicates that injury occurred before death
because inflammation cannot develop postmortem.
 Harmful in some situations
o It can also destroy normal tissues since it is non-specific.
o When it is inappropriately directed against self-tissues or not
adequately controlled, it become the cause of injury and
disease
o Such conditions are common chronic diseases such as
rheumatoid arthritis, atherosclerosis, lung fibrosis and life-
threatening hypersensitivity reactions to insect bites, drugs, Figure 2. In acute inflammation if the damaging stimulus is removed, the portion that
and toxins is abraded can be regenerated if it is skin. If there is a big chunk taken, ulceration or
o Usually include anti-inflammatory drugs to control harmful necrosis, it is no longer regeneration but healing by repair (combination of
regeneration and scar formation). If the damaging stimulus persists, it leads to chronic
sequelae of inflammation yet not interfere with its beneficial inflammation.
effects ACUTE INFLAMMATION
 A complex process that involves:  Rapid onset (typically within minutes)
i. Changes in the circulation of blood
 Short duration (hours – few days)
ii. Changes in the vessel wall permeability
iii. White cell response  Exudation of fluid and plasma proteins (edema)
iv. Release of soluble mediators  Emigration of leukocytes (i.e., neutrophils [polymorphonuclear
o These events happen simultaneously leukocytes])
 Failure to eliminate invaders will initiate the chronic inflammation
phase
 Has 3 major components (Robbins):
1. Alterations in vascular caliber increase blood flow
2. Structural changes in the microvasculature that permit
plasma proteins and leukocytes to leave the circulation

Page 1 of 44
TRANSCRIBERS: Bancod, Catindig, Magaoay, Ordoña
Acute & Chronic Inflammation & Tissue Repair

3. Emigration of the leukocytes from the microcirculation, their Inflammatory mediators such as histamine -> arteriolar dilation ->
accumulation in the focus of injury, and their activation to opening for capillary beds -> increased vascular permeability ->
eliminate the offending agent accumulation of protein-rich extravascular fluid (exudate) ->
increased blood flow and edema

GENERAL FEATURES OF INFLAMMATION


 Inflammation is normally controlled and self-limited
 defensive host response to foreign invaders and necrotic tissue,
but it is in itself capable of causing tissue damage
 Main components:
o Vascular reaction and;
o Cellular response
o *Both activated by mediators derived from plasma proteins
and various cells
FIVE Rs OR STEPS OF THE INFLAMMATORY RESPONSE:
1. Recognition of the injurious agent
2. Recruitment of leukocytes
Figure 3. Acute inflammation 3. Removal of the agent
A. Initiation 4. Regulation (control) of the response
 Results in activation of soluble mediators and recruitment of 5. Resolution (repair)
inflammatory cells to the area.  Outcome of acute inflammation is either:
 Molecules are released from the offending agent, damaged cells o Elimination of the noxious stimulus -> followed by decline of
and the extracellular matrix that alter the permeability of the reaction and repair of the damaged tissue, or
adjacent blood vessels to plasma, soluble molecules and o Persistent injury resulting in chronic inflammation
circulating inflammatory cells.
 This stereotypic, immediate response leads to rapid flooding of OVERVIEW OF ACUTE INFLAMMATION
injured tissues with fluid, coagulation factors, cytokines,
chemokines, platelets and inflammatory cells.
B. Amplification
 Depends on the extent of injury and activation of mediators such
as kinins and complement components. Additional leukocytes
and macrophages are recruited to the area.
C. Destruction of the damaging agent brings the process under
control.
 Enzymatic digestion and phagocytosis reduce or eliminate
foreign material or infectious organ- isms. At the same time,
damaged tissue components are also removed and debris is
cleared away, paving the way for repair to begin.
D. Termination of the inflammatory response
 Occurs when the offending agent is eliminated
 Resolves since the mediators are broken down and
dissipated and the leukocytes have short-life span in tissues
 Also mediated by intrinsic anti-inflammatory mechanisms
that prevent excessive damage and allow for repair and a
return to normal physiologic function.
 Repair begins during inflammation but reaches completion
usually after the injurious influence has been neutralized. It
occurs by: Figure 4.Overview of Acute Inflammation
o By replacement of tissue cells through REGENERATION
of PARENCHYMAL (bulk of the cell) cells; STIMULI FOR ACUTE INFLAMMATION
o By filling of defect with fibrous tissue (SCARRING); or, 1. Infections
most commonly, by a combination of these two o May be bacterial, viral, fungal, parasitic
processes. o Most common and medically important causes
o Important receptors for sensing microbes: Toll-like
CARDINAL SIGNS OF INFLAMMATION receptors (TLR) which when engaged, signals pathways to
1. Calor - heat/warmth produce various mediators
2. Rubor – redness 2. Trauma
3. Tumor – swelling o May be blunt and penetrating, thermal injury, irradiation,
4. Dolor – pain; caused by release of prostaglandins, toxicity from certain environmental chemicals
neuropeptides and cytokines 3. Tissue necrosis
5. Functio laesa – loss of function; Rudolf Virchow o Molecules released from necrotic cells elicit inflammation:
*Calor, rubor, and tumor – caused by increased blood flow and  Uric acid: a purine metabolite
edema  ATP: the normal energy store
 HMGB-1: a DNA-binding protein of unknown function

Page 2 of 44
TRANSCRIBERS: Bancod, Catindig, Magaoay, Ordoña
Acute & Chronic Inflammation & Tissue Repair

 DNA: when released into the cytoplasm and not o Edema denotes an excess of fluid in the interstitial tissue or
sequestered in nuclei as it should normally be serous cavities; can either be exudate or transudate
o Hypoxia, which often underlies cell injury, is also itself an o Pus is an inflammatory exudate rich in leukocytes (mostly
induce of inflammatory response as it is mediated by a neutrophils), the debris of dead cells and microbes
protein called HIF-1a (hypoxia-induced factor-1a) LIFTED FROM 2A-2016:
produced by O2-deprived cells
4. Foreign bodies (splinters, dirt, sutures)
o Can cause traumatic tissue injury or carry microbes
5. Immune reactions (hypersensitivity reactions)
o Immune system damages the individual’s own tissues
o Responses directed against self-antigens (autoimmune
diseases) – though usually, these are associated in
chronic inflammation

RECOGNITION OF MICROBES, NECROTIC


CELLS AND FOREIGN BODIES
 Microbes and dead cells must elicit some sort of "danger
signals" that distinguish them from normal tissues and mobilize
the host response
 Phagocytes, dendritic cells, and other cells, such as epithelial
cells, express receptors that are designed to sense the
presence of infectious pathogens and substances released from
dead cells
 Receptors have been called "pattern recognition receptors"
because they recognize structures (molecular patterns) that are Figure 5. Exudation and Transudation
common to many microbes or to dead cells CHANGES IN VASCULAR CALIBER AND FLOW
TWO MOST IMPORTANT FAMILIES OF RECEPTORS: ACUTE INFLAMMATION: VASCULAR EVENTS
1. Toll-like receptors (TLRs) *FROM RUBIN’S AND ROBBINS
o Located in plasma membranes and endosomes (able to 1. TRANSIENT VASOCONSTRICTION OF ARTERIOLES AT
detect extracellular and ingested microbes) THE SITE OF INJURY
o Recognize products of different types of microbes o Earliest vascular response to mild skin injury. Mediated by
o Provide defense against essentially all classes of both neurogenic & chemical mediator systems
infectious pathogens o Usually resolves within seconds to minutes
2. Inflammasome
o A cytoplasmic complex that recognizes products of dead 2. VASODILATION OF PRECAPILLARY ARTERIOLES
cells, crystals and microbial products o Occurs under the influence of no, histamine and other soluble
o Activates caspase-1, which cleaves inflammatory cytokine agents, allowing increased blood flow and expansion of the
interleukin-1β (il-1β) and il18 into its biologically active capillary bed a condition known as hyperaemia
form o Increased blood flow is responsible for redness and warmth at
sites of tissue injury
REACTIONS OF BLOOD VESSELS DURING
ACUTE INFLAMMATION 3. AN INCREASE IN THE PERMEABILITY OF THE
Exudation ENDOTHELIAL CELL BARRIER (ROBINS: OF
o Escape of fluid, proteins, and blood cells from the vascular MICROVASCULATURE) RESULTS IN EDEMA.
system into the interstitial tissue or body cavities o Disruption of this barrier is a hallmark of acute inflammation.
o Produced in mild injuries such as sunburns o Loss of fluid from intravascular compartments leads to local
Transudation stasis and plugging of dilated small vessels with erythrocytes.
o Ultrafiltration of blood plasma that results from osmotic or o Minutes to hours: the extravascular fluid is cleared through
hydrostatic imbalance without an increase in vascular lymphatics
permeability o With outpouring of protein-rich fluid into the extravascular
o Endothelial barrier remains intact and prevents the loss of tissues (Robbins)
large molecules from the vasculature o (Robbins) loss of fluid and ↑ vessel diameter →slower blood
*FROM EXUDATE TRANSUDATE flow, concentration of red cells in small vessels, and ↑
RUBIN’S viscosity of the blood → dilation of small vessels that are
Protein Conc. ↑ ↓ packed with slowly moving red cells, a condition termed
Specific >1.015 <1.015 stasis, seen as vascular congestion (localized redness)
Gravity o (Robbins) as stasis develops, blood leukocytes, principally
Inflam. Action Early Non-inflam neutrophils, accumulate along the vascular endothelium.
conditions
Cell Debris Present 4. INTRAVASCULAR STIMULATION OF PLATELETS AND
w/ neutrophils & Little to none INFLAMMATORY CELLS, WITH RELEASE OF SOLUBLE
lymphocytes MEDIATORS PRODUCED AT SITES OF INJURY,
Implications ↑ interendothelial ↑hydrostatic p STIMULATES PLATELETS AND INTRAVASCULAR
spaces ↓ osmotic p INFLAMMATORY CELLS.

Page 3 of 44
TRANSCRIBERS: Bancod, Catindig, Magaoay, Ordoña
Acute & Chronic Inflammation & Tissue Repair

o Kinins, complement and components of the coagulation RESPONSES OF THE LYMPHATIC VESSELS
cascade are activated increasing vascular permeability and  Lymphatic vessels also participate in the inflammatory response
edema  Lymphatics may become secondarily inflamed (lymphangitis) as
o Express increased levels of adhesion molecules. may the draining lymph nodes (lymphadenitis)
o Leukocytes then adhere to the endothelium, and migrate
through the vascular wall into the interstitial tissue CELLULAR EVENTS IN ACUTE INFLAMMATION
LEUKOCYTES
INCREASED VASCULAR PERMEABILITY  A critical function of inflammation is to deliver leukocytes to the
A hallmark of acute inflammation is vascular ↑ permeability site of injury and to eliminate offending agents
→escape of a protein-rich exudate into the extravascular tissue →  The most important leukocytes in typical inflammatory reactions
edema are neutrophils and macrophages
o They ingest and kill bacteria and other microbes and eliminate
necrotic tissue and foreign substances.
o They also produce growth factors that aid in repair
o HOWEVER, when strongly activated, they may induce tissue
damage and prolong inflammation because these released
products that destroy microbes and necrotic tissue can also
injure normal host tissues
 The processes involving leukocytes in inflammation are:
1. Recruitment from blood into extravascular tissues
2. Recognition of microbes and necrotic tissues, and removal of
the offending agent

LEUKOCYTE RECRUITMENT TO SITES OF INFECTION AND


INJURY
 Extravasation
o Journey of leukocytes from vessel lumen to the interstitial
Figure 6. Inflammatory mediators causing increased vascular permeability tissue
o Steps:
1. CONTRACTION OF ENDOTHELIAL CELLS RESULTING IN 1. (In the LUMEN) Margination, rolling and adhesion to
INCREASED INTERENDOTHELIAL SPACES endothelium
o Most common mechanism of vascular leakage 2. Migration across the endothelium and vessel wall
o Elicited by histamine, bradykinin, leukotrienes, the (DIAPEDESIS)
neuropeptide substance p, and many other chemical 3. Migration in the tissues toward a chemotactic stimulus
mediators (CHEMOTAXIS)
o Occurs rapidly after exposure to the mediator and usually
short-lived (15-30 mins) → called the immediate transient 1. MARGINATION, ROLLING AND ADHESION TO
response ENDOTHELIUM
o In mild injury: vascular leakage begins after a delay of 2-12 a. MARGINATION
hrs and lasts for several hours or days; this delayed prolonged  In normally flowing blood in venules, RBCS are confined to a
leakage may be caused by contraction of endothelial cells or CENTRAL AXIAL COLUMN, displacing the leukocytes towards
mild endothelial damage (eg. Late appearing sunburn) the wall of the vessel.
 In inflammation:
2. ENDOTHELIAL INJURY, RESULTING IN ENDOTHELIAL
Stasis early in inflammation (due to decreased blood flow)
CELL NECROSIS AND DETACHMENT
↓↓
o Direct damage to the endothelium is encountered in severe Changes in hemodynamic conditions (i.e. ↓ wall shear stress)
injuries, (eg. In burns or by the actions of microbes that target ↓
endothelial cells) More WBCs assume a peripheral position along the endothelial
surface
3. LEUKOCYTE-MEDIATED VASCULAR INJURY b. ROLLING
o Neutrophils that adhere to the endothelium during
 Individual then rows of leukocytes adhere transiently to the
inflammation may also injure the endothelial cells and thus
endothelium, detach and bind again, thus ROLLING on the
amplify the reaction.
vessel wall.
o Leakage starts immediately after injury and is sustained for
 Involves the interaction of adhesion molecules (selectins) in
until the damaged vessels are thrombosed or repaired.
leukocytes and the endothelium
c. ADHESION
4. INCREASED TRANSPORT OF FLUIDS AND PROTEINS,
CALLED TRANSCYTOSIS, THROUGH THE ENDOTHELIAL  Cells come to rest at some point in the endothelium where they
CELL adhere firmly
o May involve channels consisting of interconnected, uncoated  Firm adhesion is mediated by a leukocyte surface proteins
vesicles and vacuoles called the vesiculovacuolar organelle, called integrins
many of which are located close to intercellular junctions.
o Certain factors (eg. VEGF) promote vascular leakage in part
by increasing the number and perhaps the size of these
channels
Page 4 of 44
TRANSCRIBERS: Bancod, Catindig, Magaoay, Ordoña
Acute & Chronic Inflammation & Tissue Repair

ADHESION MOLECULES o Glycan-bearing cell adhesion molecule-1 (GlyCAM-1)


 Leukocyte adhesion to endothelial cells is mediated by o Mucosal addressin cell adhesion molecule-1 (MadCAM-1)
complementary adhesion molecules on two cell types whose o CD34
expression is enhanced by CYTOKINES ADDRESSINS
 Vascular addressins are mucinlike glycoproteins which include:
 CYTOKINES o GlyCAM-1
o Secreted by cells in tissues in response to microbes and other o P-selectin glycoprotein-1 (PSGL-1)
injurious agents o E-selectin ligand (ESL-1)
o This ensures that leukocytes will be recruited to the tissues o CD34
where these stimuli are present  They possess sialyl-Lewis X, which binds the lectin domain of
selectins
SELECTINS  Expressed at leukocyte and endothelium surfaces
 Responsible for early ROLLING interactions  Regulate localization of leukocyte subpopulations and are
 Molecular structure of selectins includes a chain of involved in lymphocyte activation
transmembrane glycoproteins with an extracellular lectin binding
domain. INTEGRINS
o This CALCIUM-DEPENDENT (or C-type) LECTIN binds to  Expressed by activated cells in response to:
sialylated oligosaccharides which are bound to mucin-like o Chemokines
glycoprotein backbones, specifically the SIALYL-LEWIS X o Lipid mediators
moiety on addressins o Proinflammatory molecules
 This interaction is of low affinity and has a fast off-rate, and  Possess transmembrane α and β chains arranged as
it is easily disrupted by flowing blood. Thus bound heterodimers
leukocytes bind, detach, and bind again, and thus begin to
 Participate in cell-cell interactions and cell-ECM binding
roll along the endothelial surface.
 β1, β2 and β7 integrins are involved in leukocyte recruitment
 These weak rolling interactions slow down the leukocytes
 Very late activation (VLA) molecules include VLA-4 (α4β1) on
and give them the opportunity to bind more firmly to the
leukocytes and lymphocytes that bind vascular cell adhesion
endothelium
molecule-1 (VCAM-1) on endothelial cells.
THREE TYPES OF SELECTINS  β2 (CD18) integrins form molecules by association with α-
integrin chains
 There are three types of selectins (and are specifically
expressed on the following cells): o α1β2 binds ICAM-1
o P-selectin (Platelets and endothelial cells) o αmβ2 binds ICAM-2
o E-selectin (Endothelium)  Integrins exist in a low affinity state but are converted to a high
affinity state when these cells are activated
o L-selectin (Leukocytes)
P-selectin
Chemokines produced due to injury
 Preformed and stored within WEIBEL-PALADE bodies of ↓
ENDOTHELIAL CELLS and α-granules of platelets Chemokines enter the blood vessel and binds to the
 Release is stimulates by the following mediators: endothelial cell proteoglycans
o Histamine ↓
o Thrombin Chemokines bound to proteoglycans are then displayed at
o Platelet-activating factor high concentrations on the endothelial surface
 P-selectin is rapidly transported to the cell surface, where it ↓
binds to sialyl-Lewis X on leukocyte surfaces Binding and activation of rolling leukocytes to expressed
 Preformed P-selectin can be delivered quickly to the cell chemokines
surface, allowing rapid adhesive interaction between endothelial ↓
cells and leukocytes (neutrophils, monocytes and T Activation causes conversion of VLA-4 and LFA-1 integrins on
lymphocytes) leukocytes to a high-affinity state
E-selectin
 Not normally expressed on endothelial cell surfaces but IMMUNOGLOBULINS
expression is induced by the following:  Include ICAM-1, ICAM-2 and VCAM-1
o Cytokines  All of these interact with integrins on leukocytes to mediate
o Bacterial LPS recruitment
 Mediates adhesion of neutrophils, monocytes and T  Expressed at surfaces of cytokine-stimulated endothelial cells
lymphocytes via binding to Lewis X or Lewis A and some leukocytes
L-selectin  Also expressed on certain epithelial cells such as the pulmonary
 Expressed on many types of leukocytes, mainly lymphocytes; alveolar cell
weakly expressed on neutrophils
 Originally defined as the ―homing receptor‖ for lymphocytes
 Binds lymphocytes to high endothelial venules (HEVs) in lymph
nodes and mucosal lymphoid tissues, thereby regulating their
trafficking through these tissues.
 It binds to the following:

Page 5 of 44
TRANSCRIBERS: Bancod, Catindig, Magaoay, Ordoña
Acute & Chronic Inflammation & Tissue Repair

 Occurs mainly in post-capillary venules


 Chemokines act on adherent leukocytes and stimulate the cells
to migrate through interendothelial spaces (tight and adherens
junctions) toward the site of injury or infection where the
chemokines are being produced (toward the chemical
concentration gradient)
 Involves PECAM-1 (platelet endothelial cell adhesion molecule)
or CD31
o A member of the immunoglobulin superfamily of adhesion
molecules
 The junctions between endothelial cells separate under the
influence of inflammatory mediators, intracellular signals
generated by adhesion molecule engagement and signals from
the adherent neutrophils.
o Neutrophils mobilize elastase, inducing endothelial cells
to retract and separate—a process facilitated by PMN-
elicited increases in endothelial cell intracellular calcium.
 After traversing the endothelium, the cells then migrate towards
the chemotactic gradient created by chemokines and
accumulate in the extravascular site.
 In connective tissues, leukocytes are able to adhere to the
extracellular matrix via integrins and CD44 binding to matrix
proteins—allowing them to retain at the site where they are
needed.

3. MIGRATION IN THE TISSUES TOWARD A CHEMOTACTIC


STIMULUS (CHEMOTAXIS)
 After exiting the circulation, leukocytes emigrate in tissues
Figure 7. A, Redistribution of P-selectin from intracellular stores to cell surface. B, towards the site of injury via chemotaxis
increased surface expression of selectins and ligands for integrins upon cytokine  Chemotaxis is defined as the locomotion oriented along a
activation of endothelium. C, increased binding avidity of integrins induced by chemical gradient
chemokines. (Clustering of integrins contributes to their increased binding avidity)
 Exogenous and endogenous substances may act as
chemoattractants
EXOGENOUS ENDOGENOUS
Bacterial products that Cytokines (particularly of the
contain an N- chemokine family e.g. IL-8)
formylmethionine terminal Components of the complement
amino acid system (particularly C5a)
Arachidonic acid metabolites
Some lipids
(particularly leukotriene B4)

Chemotactic agents bind to G-protein-coupled receptors



nd
Activation of 2 messengers

2+
Increase in cytosolic Ca and activation of small GMPs and
kinases
Figure 8. The multistep process of leukocyte migration through blood vessels, shown
here for neutrophils. ↓
ENDOTHELIAL LEUKOCYTE Induction of polymerization of actin
MAJOR ROLE ↓
MOLECULE MOLECULE
Increased amounts of polymerized actin at the leading edge of
P-selectin Sialyl-Lewis X Rolling the cell and localization of myosin filaments at the back
Rolling and ↓
E-selectin Sialyl-Lewis X
adhesion Leukocyte movement via filopod extension that pulls the back
GlyCAM-1, CD34 L-selectin Rolling of the cell towards the direction of extension
β2 integrins (LFA-1, Adhesion, arrest, ↓
ICAM-1 Leukocyte migration to the inflammatory stimulus in the
MAC-1) transmigration
direction of the chemokine gradient.
VCAM-1 VLA-4 (β1 integrin) Adhesion
.  Nature of leukocyte infiltrate varies with age of the inflammatory
2. MIGRATION ACROSS THE ENDOTHELIUM response and the type of stimulus.
AND VESSEL WALL (DIAPEDESIS)  In most forms of acute inflammation, neutrophils predominate in
 The next step in the process of leukocyte recruitment after the infiltrate during the first 6 to 24h and are replaced by
adhesion is migration through the endothelium—called monocytes in 24 to 48h.
transmigration or diapedesis.
Page 6 of 44
TRANSCRIBERS: Bancod, Catindig, Magaoay, Ordoña
Acute & Chronic Inflammation & Tissue Repair

 Neutrophils are the first to arrive because they:  Binding of ligands to the receptors induces migration of cells
o They are more numerous in the blood from the blood through the endothelium and production of
o Respond more rapidly to chemokines microbicidal substances by activation of respiratory burst.
o May attach more firmly to the adhesion molecules that are
rapidly induced on the endothelial cells RECEPTORS FOR OPSONINS
o They however undergo apoptosis after 24 to 48h  Leukocytes express receptors for proteins that coat microbes
 Monocytes on the other hand survive longer and may proliferate  Opsonization is the process of coating a particle to target it for
in the tissues and thus become the dominant population in ingestion (phagocytosis)
chronic inflammatory reactions.  Opsonins include:
 There are however exceptions the aforementioned pattern:  Antibodies
o Pseudomonas infections: infiltrate is dominated by  One of the most efficient ways of enhancing
neutrophils for several days phagocytosis is coating particles with IgG which are
o Viral infections: lymphocytes are the first to arrive recognized by high-affinity Fcγ receptors of phagocytes,
o Hypersensitivity reactions: eosinophils are the dominant cell called FcγRI.
type  Complement proteins
 Components of the complement system, especially C3
fragments, are also potent opsonins because they bind
to microbes and the type 1 complement receptor (CR1)
of phagocytes are able to recognize these C3 fragments
 Lectins
 Mannan-binding lectins also bind bacteria and deliver
them to leukocytes

RECEPTORS FOR CYTOKINES


 Leukocytes express receptors for cytokines that are produced in
response to microbes.
 Interferon-γ (IFN-γ) is one of the most important of these
cytokines and it is secreted by natural killer cells reacting to
microbes and by antigen-activated T lymphocytes during
Figure 9. Duration of activity of Neutrophils and Macrophages adaptive immune responses.
 IFN-γ is the major macrophage-activating cytokine.
LEUKOCYTE FUNCTIONS IN ACUTE INFLAMMATION
 Once leukocytes have been recruited to a site of infection or cell
death, they must be activated to perform their functions
 Consists of two sets of events:
1. Recognition of the offending agents
2. Activate the leukocytes to ingest and destroy the offending
agents and amplify the inflammatory reaction
 Leukocytes express several receptors that recognize external
stimuli and deliver activating signals:

RECEPTORS FOR MICROBIAL PRODUCTS


 Toll-like receptors (TLRs) recognize components of different
types of microbes.
 Different types TLRs play essential roles in cellular responses to
bacterial LPS, other bacterial proteoglycans and lipids, and
unmethylated CpG nucleotides.
 They function via receptor-associated kinases to stimulate
production of microbicidal substances and cytokines by 1. REMOVAL OF OFFENDING AGENTS
leukocytes  Recognition of microbes or dead cells by the receptors above
induces several responses in leukocytes that are referred to
G-PROTEIN COUPLE RECEPTOR under the rubric of leukocyte activation.
 Recognize short bacterial peptides containing N-  Activation results from signaling pathways triggered in
2+
formylmethionyl residues leukocytes, resulting in ↑cytosolic Ca and activation of protein
 All bacterial proteins are initiated by N-formylmethionine, thus kinase C and phospholipase A2
enabling neutrophils to detect and respond to bacterial proteins  Functional responses that are most important for microbial
 Other G-protein-coupled receptors recognize chemokines, destruction are phagocytosis and intracellular killing
breakdown products of complement such as C5a, and lipid
mediators (platelet activating factor, prostaglandins and 2. PHAGOCYTOSIS
leukotrienes), all of which are produced in response to  Involves three sequential steps:
microbes and cell injury. 1. Recognition and attachment of the particle to be ingested
2. Engulfment with subsequent vacuole formation
3. Killing or degradation of the ingested material
Page 7 of 44
TRANSCRIBERS: Bancod, Catindig, Magaoay, Ordoña
Acute & Chronic Inflammation & Tissue Repair

 Occurs most efficiently after activation of phagocytes


 It is accomplished by reactive oxygen species (ROS), reactive
nitrogen species (NOS), and as well as lysosomal enzymes
which destroy phagocytosed debris
 Potentially harmful substances are segregate from the cell’s
cytoplasm and nucleus to avoid damage to the phagocyte while
performing its function

Reactive Oxygen Species


 Microbial killing is accomplished largely by ROS, which are
generated due to rapid assembly and activation of NADPH
oxidase (phagocyte oxidase), which oxidizes NADPH and
reduces oxygen to superoxide anion.
o This is triggered by activating signals and accompanies
phagocytosis (respiratory burst)
o ROS is produced within the lysosome, protecting the cell’s
own organelles from degradation.
-
O2 anion converted to hydrogen peroxide (H2O2) through
spontaneous dismutation (superoxide dismutase)

Azurophilic granules of neutrophils release myeloperoxidase,
-
which in the presence of a halide (Cl ) converts H2O2 to
hypochlorite (OCl-2 household bleach)

Hypochlorite destroys microbes via halogenation or by lipid
peroxidation

 The H2O2-MPO-halide system is the most efficient


bactericidal system of neutrophils.
 Some O2-radicals are released extracellularly after exposure -
> tissue damage accompanying inflammation
 Antioxidant mechanisms protect against potentially harmful
O2-derived radicals
o Superoxide dismutase – detoxifies O2-radicals into H2O2
o Catalase – detoxifies H2O2
RECOGNITION AND ATTACHMENT o Glutathione peroxidase – another powerful H2O2
 Mannose receptors, scavenger receptors and receptors for detoxifier
various opsonins all function to bind and ingest microbes. o Ceruloplasmin – copper-containing serum protein
o Macrophage mannose receptor is a lectin that binds o Transferrin – Iron-containing
terminal mannose and fucose residues of glycoproteins and  Inflammatory reactions depends on the balance between
glycolipids which are typically part of microbial cell walls production and inactivation of these metabolites
 Thus, mannose receptors recognize microbes and not
host cells. Nitric Oxide Species
o Scavenger receptors are molecules that bind and mediate  Nitric oxide which is produced from arginine via action of nitric
endocytosis of oxidized LDL that can no longer react with oxide synthase also participates in microbial killing
the conventional LDL receptor o Three types:
 Macrophage scavenger receptors bind a variety of  Endothelial (eNOS)
microbes in addition to modified LDL particles.  Neuronal (nNOS)
 Both are constitutively expressed at low levels
ENGULFMENT and NO generated maintains vascular tone
 After a particle is bound to phagocyte receptors, extensions of (VASODILATOR) and acts as neurotransmitter
cytoplasm (pseudopods) flow around it and the plasma respectively.
membrane pinches off to form a phagosome that encloses the  Inducible (iNOS)
particle  Involved in microbial killing
 The phagosome fuses with a lysosomal granule which results  Induced when macrophages and neutrophils
in discharge of the granule’s contents into the phagolysosome. are activated by cytokines or microbial products
 It is dependent on polymerization of actin filaments which are o NO reacts with superoxide to generate ONOO
also involved similarly in the process of chemotaxis (peroxynitrite).
 These peroxynitrites attack lipids, proteins and
KILLING AND DEGRADATION nucleic acids of many microbes as they do with
 The final step in elimination of infectious agents and necrotic host macromolecules
cells o Overlapping actions with ROS since an experiment
observed in knockout mice lacking either phagocyte
Page 8 of 44
TRANSCRIBERS: Bancod, Catindig, Magaoay, Ordoña
Acute & Chronic Inflammation & Tissue Repair

oxidase (ROS) or iNOS are only mildly susceptible but defense because their effector mechanisms do not distinguish
those lacking both succumb to disseminated infections by between offender and host
normally harmless commensal bacteria o They suffer collateral damage
 Lysosomal enzymes, ROS and nitrogen species are able to
Lysosomal Enzymes and other Lysosomal Granules damage normal cells and vascular endothelium.
 Neutrophils and monocytes contain lysosomal granules.  The inflammatory response is inappropriately directed against
Neutrophils have two types of granules host tissues like in autoimmune diseases
o Smaller specific (secondary) granules contain lysozyme,  Sometimes, the host reacts excessively against usually
collagenase, gelatinase, lactoferrin, plasminogen activator, harmless environmental substances as in allergic reactions
histaminase and alkaline phosphatase (asthma).
o Larger azurophil (primary) granules contain MPO,  If phagocytes encounter materials that cannot easily be ingested
bactericidal factors (lysozyme, defensins), acid hydrolases, (such as immune complexes deposited on immovable flat
and neutral proteases (elastase, cathepsin G, nonspecific surface) and was unable to surround and ingest the substance
collagenases, proteinase 3) (frustrated phagocytosis), it triggers strong activation and the
 Different granule enzymes serve different functions release of large amounts of lysosomal enzymes
o Acid proteases degrade bacteria and debris within  Some phagocytosed substances (urate crystals) may damage
phagolysosomes which are acidified by membrane-bound the membrane of the phagolysosome and also lead to the
proton pumps release of lysosomal contents
o Neutral proteases degrade various extracellular
components such as collagen, basement membrane, fibrin, OTHER FUNCTIONAL RESPONSES OF ACTIVATED
elastin, and cartilage -> inflammatory process + tissue LEUKOCYTES
destruction  Macrophages produce cytokines that can either amplify or limit
 It can also cleave complement proteins directly yielding inflammatory reactions
anaphylatoxins and kinin-like peptide  These produce growth factors that stimulate:
o Neutrophil elastase degrade virulence factors of bacteria
o Endothelial cell and fibroblast proliferation
and combat bacterial infections
o Synthesis of collagen and enzymes that remodel connective
 Controlled by system of antiproteases in serum and tissue fluids.
tissues
Major inhibitor of neutrophil elastase is α1-antitrypsin
 These drive the process of repair
o Deficiency of which leads to sustained action of leukocyte
 ―Classically activated‖ macrophages
proteases
 Microbial killing also accurs via action of other substances in o Respond to microbial products and T-cell cytokines (i.e. IFN-
leukocyte granules. γ)
o Other granules include:  ―Alternatively activated‖ macrophages
 Defensins (toxic to microbes) o Respond to IL-4 and IL-13
 Cathelicidins (antimicrobials in neutrophils) o Mainly involved in tissue repair and fibrosis
 Lysozymes (hydrolyzes muramic acid-N-
acetylglucosamine bond in the peptidoglycan coat of
bacteria
 Lactoferrin
 Major basic protein (released by eosinophils, cytotoxic to
parasites)
 Bactericidal/permeability increasing protein (binds to
endotoxin, important against gram negative bacteria)

NEUTROPHIL EXTRACELLULAR TRAPS (NETs)


 Extracellular fibrillar networks that provide a high concentration
of antimicrobial substances at sites of infection and prevent the
spread of the microbes by trapping them in to fibrils
 Produced by the neutrophils in response to
o Infectious pathogens
o Inflammatory mediators  T lymphocytes, cells of adaptive immunity, also contribute to
acute inflammation since they produce the cytokine IL-17 (TH17
 In the process, nuclei of neutrophils are lost, leading to death of
cells)
the cells
o These induce secretions of chemokines that recruit other
 These are also detected in the blood during sepsis and their
leukocytes
formation is believed to be dependent on platelet activation
o In its absence, ↑ fungal and bacterial infection susceptibility
 Nuclear chromatin in the NETs are postulated to be the source o Skin abscess (―cold abscesses‖) which lacks the classic
of nuclear antigens particularly in Systemic Lupus features of acute inflammation such as warmth and redness
Erythematosus (SLE) in which individuals react against their
own DNA and nucleoproteins DEFECTS IN LEUKOCYTE FUNCTION
INHERITED DEFECTS IN LEUKOCYTE ADHESION
RELEASE OF LEUKOCYTE PRODUCTS AND LEUKOCYTE-
 Leukocyte Adhesion Deficiency-1 (LAD1)
MEDIATED INJURY
 The mechanisms by which leukocytes damage normal tissues
o Defect in the biosynthesis of the β2 chain shared by LFA-1
and Mac-1 integrins
are the same as the mechanisms involved in antimicrobial

Page 9 of 44
TRANSCRIBERS: Bancod, Catindig, Magaoay, Ordoña
Acute & Chronic Inflammation & Tissue Repair

 Leukocyte Adhesion Deficiency-2 (LAD2)


o Absence of sialyl-Lewis X ligand for E- and P-selectins
o Due to a defect I fucosyl transferase, the enzyme that
attaches fucose moieties to protein backbones.

INHERITED DEFECTS IN PHAGOLYSOSOME FUNCTION


 Chediak-Higashi Syndrome
o Autosomal recessive condition characterized by defective
fusion of phagosomes and lysosomes in phagocytes
o Main leukocyte abnormalities are neutropenia, defective
degranulation and delayed microbial killing
o WBCs contain giant granules
o The genetic defect lies in the LYST gene, which regulated
lysosomal trafficking

INHERITED DEFECTS IN MICROBICIDAL ACTIVITY


 Chronic Granulomatous Disease
o Defects in bacterial killing, thus rendering patients
susceptible to recurrent bacterial infection
o Results from inherited defects in the genes encoding for
phagocyte oxidase/MPO.
o This often leads to collections of macrophages that wall of
microbes, forming granulomas
1. The most important mediators of acute inflammation are
ACQUIRED DEFICIENCIES vasoactive amines, lipid products (prostaglandins,
 Most frequent cause of leukocyte defects is bone marrow leukotrienes), cytokines (including chemokines), and
suppression products of complement activation.
o Leads to decreased WBC production o These induce various components of the inflammatory
o Seen in: response by distinct mechanisms
 Patients who underwent cancer therapy (radiation and  Inhibiting each has been therapeutically beneficial
chemotherapy) 2. Mediators are either secreted by cells or generated from
 Tumors that compromise the marrow space plasma proteins.
 May arise in the marrow o Cell-derived mediators
 Metastasis  Sequestered in intracellular granules
 Released by granule exocytosis or synthesized de
TERMINATION OF THE ACUTE INFLAMMATORY RESPONSE novo in response to stimuli
 Inflammation partly declines because mediators of inflammation o Major cell types that produce mediators of acute
are produced: inflammation are the sentinels that detect invaders and
o In rapid bursts damage in tissues
o Only as long as the stimulus persists  Macrophages
 Dendritic cells
o Have short half-lives
 Mast cells
o Degraded after their release  Also platelets, neutrophils, endothelial cells, most
 Neutrophils also have short half-lives and due via apoptosis a epithelia
few hours after leaving the blood
o Plasma-derived mediators
 Stop signals are also triggered to actively terminate the reaction  Produced mainly in the liver
as the inflammation develops  Present in the circulation as inactive precursors and
o Switching in the type of arachidonic acid metabolite from pro- must be activated via proteolytic cleavages
inflammatory leukotrienes to anti-inflammatory lipoxins 3. Active mediators are produced only in response to
o Liberation of anti-inflammatory cytokines including TGF-β various stimuli
and IL-10 from macrophages and other cells o Stimuli include:
o Production of anti-inflammatory lipid mediators called  Microbial products
resolvins and protectins  Substances released by necrotic cells
o Neural impulses (cholinergic discharge) that inhibits TNF  Proteins of the complement systems, coagulation
production by macrophages system and kinins
o Initiating stimulus  microbes or dead tissues
MEDIATORS OF INFLAMMATION  Ensures that inflammation is normally triggered only
 Substances that initiate and regulate inflammatory reactions when and where it is needed
4. Most of the mediators are short-lived
o They quickly decay, or are inactivated by enzymes,
scavenged, or inhibited
o There is a built-in control system that checks and balances
mediator actions

Page 10 of 44
TRANSCRIBERS: Bancod, Catindig, Magaoay, Ordoña
Acute & Chronic Inflammation & Tissue Repair

5. One mediator can stimulate the release of other ARACHIDONIC ACID METABOLITES
mediators
o TNF can stimulate IL-1 and other chemokine production
by acting on endothelial cells
o Products of complement activation stimulate the release of
histamine

6. Mediators vary in their range of cellular targets


o Can act on one or a few target cell types
o Can have diverse targets
o May have differing effects on different cell types

VASOACTIVE AMINES: HISTAMINE & SEROTONIN


 Stored as preformed molecules in cells, thus they are among the
first mediators to be released during inflammation
 So named vasoactive because they have important actions on
blood vessels

HISTAMINE
 Richest source  mast cells
o Mast cells are normally present in the connective tissue
adjacent to blood vessels
o Also found in blood basophils and platelets
 Stored in mast cell granules
 Released by mast cell degranulation in response to:
o Physical injury (trauma, cold, heat) Prostaglandins and Leukotrienes
o Antibody binding to mast cells (immediate allergic reactions)
 Produced from arachidonic acid present in membrane
o Due to anaphylatoxins (C3a and C5a) phospholipids
o Histamine-releasing proteins from leukocytes  Stimulate vascular and cellular reactions in acute inflammation
o Neuropeptides (substance P)  When cells are activated by diverse stimuli (microbial products,
o Cytokines (IL-1, IL-8) mediators), AA is rapidly converted to produce prostaglandins
 Causes vasodilation of arterioles and increases the and leukotrienes
perbeability of venules
 Also causes contraction of some smooth muscles Arachidonic Acid (AA)
 Principal mediator of the immediate transient phase of increased
 20-carbon polyunsaturated fatty acid (5,8,11,14-eicosatetraenoic
vascular permeability
acid)
o This produces interendothelial gaps in venules  Derived from dietary sources or by conversion from the essential
 Vasoactive effects are mediated via binding to H1 receptors on fatty acid LINOLEIC ACID
microvascular endothelial cell
 Does not occur free in the cell but is normally esterified in
 Antihistamine drugs against inflammatory reactions such as
membrane phospholipids
allergies are H1 receptor antagonists that bind and block the
receptor
 Mechanical, chemical and physical stimuli or other mediators
(C5a) release AA from membrane phospholipids via action of
phospholipases (mainly phospholipase A2)
SEROTONIN (5-HYDROXYTRYPTAMINE)
 Preformed vasoactive mediator present in platelets and certain Phospholipase A2 (PLA2)
neuroendocrine cells of the GIT; in mast cells in rodents but not
 Activation includes an increase in cytosolic Ca2+ and kinase
humans
activation
 Primary function  neurotransmitter in the GI tract
 Release by platelets is stimulated when they aggregate after Eicosanoids
contact with:
 AA derived mediators
o Thrombin
 Derived from 20-C fatty acids
o Collagen
 Synthesized by 2 major classes of enzymes: cyclooxygenase
o ADP
and lipoxygenase
o Ag-Ab complexes
 Cyclooxygenases or COX  generates prostaglandins via
 Also a vasoconstrictor (9 ed), but importance of this action in
th
cyclooxygenation)
inflammation is unclear
 Also results in increased vascular permeability (8 ed)
th  Lipoxygenases  produces leukotrienes and lipoxins via
lipooxygenation
 Bind to G protein-coupled receptors; can mediate virtually every
step of inflammation
 Inhibited by corticosteroids (8th ed)

Page 11 of 44
TRANSCRIBERS: Bancod, Catindig, Magaoay, Ordoña
Acute & Chronic Inflammation & Tissue Repair

 Induction of a PLA2 inhibitor and blockage of the release of AA  Vasodilation and increases permeability of post-capillary
in inflammatory cells venules
o Potentiates edema formation
 PGD2
o Chemoattractant for neutrophils
 PGE2
o Hyperalgesic (makes skin hypersensitive to painful stimuli
such as in intradermal injection)
o Involved in cytokine-induced fever during infections

PGF2a
 Stimulates contraction of uterine and bronchial smooth muscle
and small arterioles
PROSTAGLANDINS (PGs)
 Produced by: LEUKOTRIENES (LT)
o Mast cells
 Produced by leukocytes and mast cells by the action of
o Macrophages lipoxygenase enzymes
o Endothelial cells  Involved in vascular and smooth muscle reactions and leukocyte
 Involved in vascular and systemic reactions of inflammations recruitment
 Produced by two enzymes:  There are three different types of lipoxygenases but 5-
o The constitutively expressed COX-1 LIPOXYGENASE is the most predominant in neutrophils
o Inducible COX-2 o Converts AA to 5-hydroxyeicosatetraenoic acid
 COX-1 (chemotactic for neutrophils and leukotriene precursor)
o Produced in response to inflammatory stimuli and is also  LTB4
constitutively expressed in most tissues o Potent chemotactic agent and activator of neutrophils
o May serve a homeostatic function o Causes aggregation and adhesion of cells to venular
 COX-2 endothelium
o Induced by inflammatory stimuli and thus generates o Causes ROS generation
prostaglandins that are involved in inflammatory reactions o Causes lysosomal enzyme release
o Low or absent in most normal tissues  LTC4, LTD4, LTE4 (Cysteinyl containing leukotrienes)
 Divided into series based on structural features coded by a letter o Causes intense vasoconstriction
(PGD, PGE, PGF, PGG, PGH) and a subscript numeral (1 or 2) o Causes bronchospasm
which indicates the number of double bonds in the compounds o Increases venular permeability
 Most important for inflammation:  Vascular leakage is restricted to venules
o PGE2  LTs are more potent than histamine in increasing vascular
o PGD2 permeability and causing bronchospasm
o PGF2a
o PGI2 (prostacyclin) LIPOXINS
o TxA2 (thromboxane A2)  Also derived from AA by the lipoxygenase pathway
 Involved in the pathogenesis of pain and fever in inflammation
 Suppress inflammation unlike PGs and LTs
THROMBOXANE (TXA2) o Inhibits recruitment of leukocytes
o Inhibit neutrophil chemotaxis and adhesion to
 In platelets because thromboxane synthetase is found in endothelium
platelets  Prostaglandins and leukotrienes are required for the cellular
 Potent plagelet-aggregating agent and vasoconstrictor biosynthesis of lipoxins
 Is unstable and rapidly converts to its inactive form TxB  Leukocytes, particularly neutrophils, produce intermediates in
lipoxin synthesis and platelets that interact with these leukocytes
PROSTACYCLIN (PGI2) convert them to lipoxins
 Vascular endothelium lacks thromboxane synthetase but  There exists an inverse relationship between lipoxin and
possesses prostacyclin synthetase, forming prostacyclin and its leukotriene production
stable end product PGF1α
 Actions:
o Vasodilator
o Potent inhibitor of platelet aggregation
o Potentiates the permeability-increasing and chemotactic
effects of other mediators

PGD2 AND PGE2


 Major prostaglandins made by mast cells
 PGE2 is more widely distributed
Page 12 of 44
TRANSCRIBERS: Bancod, Catindig, Magaoay, Ordoña
Acute & Chronic Inflammation & Tissue Repair

OTHER APPROACHES
 Modify intake and content of dietary lipids
 Increase consumption of fish oil
o Polyunsaturated fatty acids are poor substrates for
conversion to active metabolites by COX and
lipoxygenase pathways
o But are better substrates for production of anti-
inflammatory lipid products

CYTOKINES AND CHEMOKINES


CYTOKINES
 Proteins produced by many cell types
o Principally activated lymphocytes, macrophages,
dendritic cells
o Also from endothelial, epithelial, connective tissue cells
 Mediate and regulate immune and inflammatory reactions
 NOTE: growth factors that act on epithelial and mesenchymal
cells are NOT grouped under cytokines

PHARMACOLOGIC INHIBITORS OF PROSTAGLANDINS AND


LEUKOTRIENES
CYCLOOXYGENASE INHIBITORS
 Include aspirin and other nonsteroidal anti-inflammatory drugs
(NSAIDs) such as ibuprofen
 Inhibit both COX-1 and COX-2
 Inhibit prostaglandin synthesis (hence their efficacy in treating
pain and fever)
 MOA of aspirin:
o Irreversibly acetylating and inactivating COX
 Selective COX-2 inhibitors
o Newer class of drugs
o 200-300 fold more potent in blocking COX-2 than COX-1
o May increase risk of CV and cerebrovascular events, possibly
because they impair endothelial cell production of prostacyclin
but leave intact the COX-1 mediated production of
thromboxane A2
o May tilt balance towards thromboxane and promote vascular
thrombosis
 More pharmacologic focus is on inhibiting COX-2
o COX-1 is believed to be responsible in the production of
prostaglandins that are involved in both inflammation and
homeostatic functions in the kidneys, cytoprotection in the GI
tract
o COX-2 generated prostaglandins that are involved only in TUMOR NECROSIS FACTOR (TNF)
inflammatory reactions AND INTERLEUKIN-1 (IL-1)
 Serve critical roles in leukocyte recruitment
LIPOXYGENASE INHIBITORS
o Promotes adhesion of leukocytes to endothelium and
 5-lipoxygenase is NOT affected by NSAIDs their migration through vessels
o useful in the treatment of asthma (e.g., Zileuton)  Two of the major cytokines that mediate inflammation
 Produced mainly by activated macrophages and dendritic cells
CORTICOSTEROIDS  TNF is also produced by T lymphocytes and mast cells
 Broad-spectrum anti-inflammatory agents  IL-1 is produced by some epithelial cells as well
 Reduce transcription of genes encoding COX-2, phospholipase  Secretion is stimulated by:
A2, proinflammatory cytokines (IL-1, TNF) and iNOS o Endotoxin and other microbial products
o Immune complexes
LEUKOTRIENE RECEPTOR ANTAGONISTS o Foreign bodies
 Block leukotriene receptors o Physical injury
 Prevent actions of leukotrienes o Inflammatory stimuli
 Useful in treatment of asthma (e.g., Montelukast)  Production of TNF is induced by signals through TLRs and
other microbial sensors

Page 13 of 44
TRANSCRIBERS: Bancod, Catindig, Magaoay, Ordoña
Acute & Chronic Inflammation & Tissue Repair

 Synthesis of IL-1 is stimulated by the same signals but the INTERFERON-γ (IFN-γ)
generation of its biologically active form is dependent on the 8 ed
th
inflammosome
 Also a potent stimulus for macrophage activation and cyrtokine
 ACTIONS:
production
o Endothelial activation
 Produced by a subset of T lymphocytes
 Increased expression of endothelial adhesion
 Synthesized also by NK cells in the primary host response
molecules (E- and P-selectins and ligands for
leukocytte integrins)  When exposed to IL-12 and TNF-α, NK cells are activated to
 Increased production of various mediators produce IFN-γ
 Increased procoagulant activity of the endothelium
o Activation of leukocytes and other cells CHEMOKINES
 TNF augments responses of neutrophils bacterial  Small proteins that act primarily as chemoattractants for
endotoxins specific leukocyte types
 Stimulates microbicidal activity of macrophages by  They regulate leukocyte trafficking in inflammation and
inducing production of NO immunity
 IL-1 activates fibroblasts to synthesize collagen  Most important chemotactic factors for neutrophils: (8 ed)
th

and stimulates TH17 responses o C5a


o Systemic acute-phase responses o N-formylated peptides
 Include fever o Products of arachidonic acid metabolism, especially LTB4
 TNF and IL-1 are also implicated in sepsis,
o Chemokines
resulting from disseminated bacterial infection
 Classified into FOUR major groups, according to the
 TNF regulares energy balance by promoting lipid
arrangement of cysteine (C) residues
and protein mobilization and decreasing appetite
 Sustained TNF production  leads to cachexia  Mediate their activities by binding to seven-transmembrane G
protein-coupled receptors

th
Inflammosome (8 ed)
o Usually exhibit overlapping ligand specificities
o Controls IL-1 production
o Leukocytes generally express more than one receptor
o Activates proteases (caspases) which cleave inactive IL-1 type
precursors into the biologically active cytokine
o CXCR-4 and CCR-5
o Mutations cause inherited autoinflammatory syndromes
 Act as coreceptors for a viral envelope glycoprotein
(e.g. Mediterranean fever)
of HIV-1
 These mutations result to an unregulated IL-1
 Involved in binding and entry of the virus (HIV) into
production
cells
 IL-1 antagonists are used to treat this disorders
 Two main functions:
o In acute inflammation
 Production is induced by microbes and other
stimuli
 Stimulate leukocyte attachment to endothelium by
acting on leukocytes to increase affinity of integrins
 Stimulate migration (chemotaxis) of leukocytes in
tissues to the site of infection or tissue damage
o Maintainance of tissue structure
 Some are produced constitutively in tissues
(homeostatic chemokines)
 Organize various cell types in different anatomic
regions of tissues, such as T and B lymphocytes in
discrete areas of spleen and lymph nodes

C-X-C CHEMOKINES
 Also known as α chemokines
 Have 1 amino acid separating the first 2 of the 4 conserved
cysteine residues
 Act primarily on neutrophils
 IL-8 is an example of an α chemokine
 Secreted by activated macrophages, endothelial cells and
other cell types
 Causes activation and chemotaxis of neutrophils (with limited
activity on monocytes and eosinophils)
 Most important inducers of IL-8 include microbial products and
other cytokines (mainly IL-1 and TNF)

C-C CHEMOKINES
 Also known as β chemokines

Page 14 of 44
TRANSCRIBERS: Bancod, Catindig, Magaoay, Ordoña
Acute & Chronic Inflammation & Tissue Repair

 Have the first 2 conserved cysteine residues adjacent o Both granule types can fuse with phagocytic vacuoles
 Include: containing engulfed material
o Monocyte chemoattractant protein (MCP-1) o Granules may also be released into extracellular space
o Eotaxin (selectively recruits eosinophils)  Neutral proteases
o Macrophage inflammatory protein-1α (MIP-1α) o Capable of degrading various extracellular components,
o RANTES (regulated and normal T0cell expressed and such as collagen, basement membrane, fibrin, elastin, and
secreted) cartilage= tissue destruction (w/ inflammatory processes)
 Generally attract monocytes, eosinophils, basophils and o Cleave C3 and C5 complement proteins directly, thus
lymphocytes but not neutrophils releasing anaphylatoxins
 Neutrophil elastase
C CHEMOKINES o Degrade virulence factors of bacteria
 Also known as γ chemokines o Combat bacterial infections
 E.g., lymphotactin  Monocytes and macrophages
 Lack the first and third of the 4 coserved cysteines o Contain acid hydrolases, collagenase, elastase,
 Relatively specific for lymphocytes phospholipase, and plasminogen activator
 Lysosomal enzymes can potentiate further inflammation and
tissue damage during the initial leukocytic infiltration
CX3C CHEMOKINES
 α1-antitrypsin
 The only member is fractalkine o Major inhibitor of neutrophil elastase
 3 amino acids between 2 cysteines o Deficiency leads to sustained action of leukocyte
 Fractalkine exists in two forms: proteases
1. Cell surface-bound protein  α2-Macroglobulin
o Can be induced on endothelial cells by inflammatory o Antiprotease found in serum and various secretions
cytokines
o Promotes strong adhesion of monocytes and T cells THE COMPLEMENT SYSTEM
2. Soluble form
o Derived by proteolysis of the above form
o Potent chemoattractant activity for monocytes & T cells

OTHER CYTOKINES IN ACUTE INFLAMMATION


 Interleukin-6 (IL-6)
o Produced by macrophages and other cells
o Involved in local and systemic reactions
 Interleukin-17 (IL-17)
o Produced mainly by T-lymphocytes
o Promotes neutrophil recruitment

LYSOSOMAL CONSTITUENTS OF LEUKOCYTES


th
From 2A 2017 trans; Robbins 8 edition
 Neutrophils and monocytes contain lysosomal granules  Collection of soluble proteins and membrane receptors
 These granules may contribute to the inflammatory response concerned in host defense against microbes and in pathologic
when released inflammatory reactions
 NEUTROPHILIC GRANULES  Consists of >20 proteins
o Specific or Secondary granules  Functions in both innate and adaptive immunity for defense
 Smaller against microbial pathogens
 Contain:  Activation involve several cleavage products of complement
 Lysozyme proteins
 Collagenase o Cause increased vascular permeability, chemotaxis and
 Gelatinase opsonization
 Lactoferrin  Proteins are present in inactive forms in plasma
 Plasminogen activator o Most are activated to become proteolytic enzymes that
 Histaminase degrade other complement proteins
 Alkaline phosphatase  This forms a cascade capable of tremendous
o Primary or Azurophilic granules amplification
 Contain:  CRITICAL STEP:
 Myeloperoxidase o Proteolysis of the third and most abundant component, C3
 Lysozyme  Three ways in which C3 may be cleaved:
 Defensins 1. Classical pathway (triggered by fixation of C1 to an Ag-Ab
 Acid hydrolases complex)
 Neutral proteases (elastase, cathepsin G, non-
specific collagenases, proteinase 3)

Page 15 of 44
TRANSCRIBERS: Bancod, Catindig, Magaoay, Ordoña
Acute & Chronic Inflammation & Tissue Repair

2. Alternative pathway (triggered by microbial surface  They thereby increase vascular permeability and cause
molecules such as endotoxin, LPS, and venom in the vasodilation
absence of an Ab)  They are therefore called anaphylatoxins because their
3. Lectin Pathway (plasma mannose-binding lectin binds to effects are similar to mast cell mediators involved in
carbohydrates on microbes, directly activating C1) anaphylaxis
 C3a, C5a, C5a = anaphylatoxins
 All three pathways lead to the formation of an active enzyme  C5a
called the c3 convertase, which splits c3 into c3a and c3b o Also a powerful chemotactic agent for neutrophils,
monocytes, eosinophils, and basophils
Formation of C3 convertase o Also activates the lipoxygenase pathway of AA metabolism
↓ in neutrophils and monocytes, causing further release of
C3 cleaved into C3a and C3b inflammatory mediators

C3a is released; C3b covalently attaches to the cell or molecule
where complement is being activated

C3b binds to previously generated fragments to form
C5 convertase

Cleavage of C5 to C5a and C5b

C5a is released; C5b attaches to cell surface

C5b binds to late components C6 - C9

Formation of the MAC (multiple C9 molecules)

OPSONIZATION AND PHAGOCYTOSIS


 C3b and its cleavage product iC3b (inactive C3b) acts as
opsonins when fixed to a microbial cell wall
 Opsonins promote phagocytosis by neutrophils and
macrophages which bear cell surface receptors for the
complement fragments

CELL LYSIS
 Deposition of the MAC on cells makes them permeable to
water and ions which results to death (lysis)
 For the killing of microbes with thin cell walls (Neisseria)

REGULATION OF THE COMPLEMENT SYSTEM


 Activation of complement is tightly controlled by cell-associated
and circulating regulatory proteins
 Prevents healthy tissues from being injured at sites of
complement activation
 Regulatory proteins can be overwhelmed when large amounts
of complement are deposited on host cells and in tissues
o Autoimmune diseases
 Four main mechanisms: (from 2A 2017 trans)
1. SPONTANEOUS DECAY
o C4b2a and C3bBb and their cleavage products C3b and
C3b decrease by decay

FUNCTIONS OF THE COMPLEMENT SYSTEM 2. PROTEOLYTIC INACTIVATION


o Factor 1 is an inhibitor of C3b and C4b
INFLAMMATION o Serum carboxypaptidase N (SCPN) cleaves the arginine of
anaphylatoxins which markedly decreases their biological
 C3a, C5a and to a lesser extent, C4a stimulate histamine activity
release from mast cells
Page 16 of 44
TRANSCRIBERS: Bancod, Catindig, Magaoay, Ordoña
Acute & Chronic Inflammation & Tissue Repair

NEUROPEPTIDES
3. BINDING OF ACTIVE COMPONENTS
 Secreted by sensory nerves and various leukocytes
o C1 esterase inhibitor binds C1r and C1s, forming an  Play a role in the initiation nd propagation of an inflammatory
irreversibly inactive complex
response
o Factor H and C4b-binding protein complex with C3b and  Examples: Substance P and Neurokinin A
C4b respectively
o Produced in the central and peripheral nervous systems
 This enhances their susceptibility to proteolytic
cleavage by factor 1.  Substance P
o Fibers containing substance P are prominent in the lung
4. CELL MEMBRANE-ASSOCIATED MOLECULES and GIT
o Decay-accelerating factor (DAF) o Functions:
 Breaks down the alternative pathway C3  Transmission of pain signals
convertase  Regulation of blood pressure
 Stimulation of secretion by endocrine cells
o Protectin (CD59)
 Binds membrane-associated C4b and C3b,  Increasing vascular permeability
promoting its inactivation by factor 1  Some sensory neurons can also produce calcitonin-related
 It also prevents MAC formation gene product
o Links sensing of painful stimuli to development of
 C1 inhibitor (C1 INH) protective host responses.
o Blocks activation of C1, the first protein of the classical
pathway COAGULATION AND KININ SYSTEMS
th
o Inherited deficiency of this inhibitor  hereditary *from 2A 2017 trans; Robbins 8 edition
angioedema  Inflammation and blood clotting are intertwined, with each
 Activation of complement by antibodies or Ag-Ab complexes promoting the other
deposited on host cells and tissues is an important  The intrinsic clotting pathway is a series of plasma proteins
mechanism of cell and tissue injury activated by Hageman Factor (Factor XII)
 Inherited deficiencies of complement proteins cause
increased susceptibility to infections HAGEMAN FACTOR (FACTOR XII)
 Activated when it comes in contact with:
OTHER MEDIATORS OF INFLAMMATION
o Collagen
PLATELET ACTIVATING FACTOR (PAF) o Basement membrane as a result of endothelial damage
 Phospholipid-derived mediator o Proteolytic enzymes
 Actions: o Bacterial LPS
o Causes platelet aggregation o Foreign materials
o Has multiple inflammatory effects  After activation, its conformation changes to factor XIIa which
o Can cause vasoconstriction and bronchoconstriction leads to:
o Vasodilation and increased venular permeability at 1. CONVERSION OF PLASMINOGEN TO PLASMIN
extremely low concentrations o Induces fibrinolysis
 Potency is 100 to 10,000 x than that of histamine o Fibrin degradation products augment vascular permeability
o Also causes increased leukocyte adhesion to in the skin and lung
endothelium o Plasmin also cleaves complement components, generating
 By enhancing integrin-mediated leukocyte binding active products like C3a and C5a
o Leukocyte chemotaxis, degranulation and oxidative burst 2. CONVERSION OF PREKALLIKREIN TO KALLIKREIN
o Boosts synthesis of other mediators (particularly o Kallikrein cleaves high molecular weight kininogen, which
eicosanoids) then produces kinins
 Elaborated by platelets, basophils, mast cells, neutrophils, 3. ACTIVATION OF THE ALTERNATIVE COMPLEMENT
macrophages, endothelial cells PATHWAY
o Both secreted and cell bound forms 4. ACTIVATION OF THE COAGULATION SYSTEM

PRODUCTS OF COAGULATION KININS


 previous studies  inhibition of coagulation decreased  Vasoactive peptides derived from kininogens by actions of
inflammatory reaction to some microbes kallikreins
 Protease-activated receptors (PARs)
o Activated by thrombin Factor XIIa converts prekallikrein to kallikrein
o Expressed on platelets and leukocytes ↓
o Major role: platelet activation during clotting Kallikrein cleaves high molecular weight kininogen (HMWK) to
 Virtually all forms of injury that lead to clotting also induce produce BRADYKININ
inflammation  Bradykinin causes: (similar to histamine)
 Inflammation causes changes in endothelial cells that o Increased vascular permeability
increase likelihood of abnormal clotting o Contraction of smooth muscle
o Dilation of blood vessels

Page 17 of 44
TRANSCRIBERS: Bancod, Catindig, Magaoay, Ordoña
Acute & Chronic Inflammation & Tissue Repair

o Pain when injected into the skin OUTCOMES OF ACUTE INFLAMMATION


 The actions of bradykinin are short-lived due to its inactivation 1. COMPLETE RESOLUTION
via kininase o Restoration of the injured site to normal after the
 Kallikrein itself is also a potent activator of Hageman factor, inflammatory reaction has succeeded in eliminating the
allowing for autocatalytic amplification of the initial stimulus offending agent
 The most significant function of kinins is their ability to amplfy o Is the usual outcome when injury is limited or short-lived or
inflammatory responses by stimulating local tissue cells to when there has been little tissue destruction that the
generate additional mediators such as: damaged cells can regenerate
o Prostanoids o Involves removal of debris and microbes by macrophages
o Cytokines (TNF-α and ILs) and resorption of edema fluid by lymphatics
o Nitric oxide
o Tachykinins 2. HEALING BY CONNECTIVE TISSUE REPLACEMENT
 Bradykinin has been implicated as a mediator in some forms (SCARRING/FIBROSIS)
of allergic reaction, such as anaphylaxis o Occurs after substantial tissue destruction
o Involves tissues that are incapable of regeneration or
FIBRINOLYTIC SYSTEM when there is abundant fibrin exudation in tissue or serous
cavities that can’t be adequately cleared
 This cascade counterbalances clotting by fibrin cleavage,
solubilizing the clot
o Connective tissue grows into the area of damage or
exudate, converting it into a mass of fibrous tissue in a
 Functions of the fibrinolytic system:
process called organization
o Lysing of fibrin clots o Tissue is destroyed to some extent and must be repaired
o Cleaves complement protein C3 to C3 gragments o Damaged cells are replaced, and the framework of the
o Degrades fibrin to form fibrin split products injured tissue is repaired as an ingrowth of cells produces
 These may have permeability inducing properties connective-tissue fibers and new blood vessels
o Scar tissue replaces large areas of tissue destruction
o Scarring may be so severe that function is seriously
disturbed

3. PROGRESSION OF RESPONSE TO CHRONIC


INFLAMMATION
o Acute to chronic transition occurs when acute
inflammatory response can’t be resolved as a result of:
 Persistence of the injurious agent
 Interference with the normal process of healing

*following photos are from 2A 2017 trans

Page 18 of 44
TRANSCRIBERS: Bancod, Catindig, Magaoay, Ordoña
Acute & Chronic Inflammation & Tissue Repair

MORPHOLOGIC PATTERNS OF ACUTE INFLAMMATION


 The morphologic hallmarks of all acute inflammatory reactions
are:
o Dilation of small blood vessels
o Slowing of blood flow
o Accumulation of leukocytes and fluid in extravascular
tissue
 Special patterns are often superimposed on these three
depending on the severity of the reaction, its cause and the
particular tissue and site involved.
Acute inflammation of face with superficial necrosis of skin. Crust of dried exudate
(scabs) have formed on skin surface. SEROUS INFLAMMATION
 Marked by exudation of cell-poor fluid into spaces created by
cell injury or into body cavities lined by peritoneum, pleura, or
pericardium
 Typically, fluid is not infected by destructive organisms and
does not contain large numbers of leukocytes
 Fluid may be derived from plasma (from increased vascular
permeability) or from secretions of mesothelial cells (from local
irritation)
 This accumulation of fluid is called an EFFUSION
Extensive tissue destruction of lower lip, which is covered with inflammatory exudate. o Examples: skin blister resulting from a burn or viral
Child chewed an electric light cord, exposing bare wire, and sustained a severe infection
electrical burn of lip.

Marked scarring after the healing of a severe burn, which has restricted motion of
Extensive burn with marked leakage (extravasation) of fluid into the burned area
neck and arms. Skin grafting was required to improve function.
leading to formation of large blisters. (Photo from 2A 2017 trans)

CLINICAL PRESENTATION OF ACUTE INFLAMMATION


*from 2A 2017 trans

Marked swelling of ear caused by acute inflammation Morphologic feature: Serous (watery) exudates
Morphologic feature: Swelling - Consist mostly of fluid and small amounts of protein and WBCs
- Occurs because of the leakage of plasma from the dilated and - Often caused by allergic reactions or burn
more permeable vessels -
- Causes the volume of fluid in the inflamed tissue to increase FIBRINOUS INFLAMMATION
- Tenderness and pain are secondary to irritation of sensory  With greater increase in vascular permeability, fibrinogen
nerve endings at the site of the inflammatory process molecules pass the vascular barrier and fibrin is formed and
deposited in the extracellular space

Page 19 of 44
TRANSCRIBERS: Bancod, Catindig, Magaoay, Ordoña
Acute & Chronic Inflammation & Tissue Repair

 A fibrinous exudate develops when the vascular leaks are PURULENT/SUPPURATIVE INFLAMMATION; ABSCESS
large or there is a local procoagulant stimulus  Characterized by the production of large amounts of pus or
 Characteristic of inflammation in the linings body cavities purulent exudate consisting of:
(meninges, pericardium, pleura) o Neutrophils
 Fibrin appears histologically as an eosinophilic meshwork of o Liquefactive debris of necrotic cells
threads or as an amorphous coagulum o Edema fluid
 These exudates may be removed via fibrinolysis and clearing  Most frequent cause is infection with bacteria that cause
of debris by macrophages liquefactive tissue necrosis, such as staphylococci
 If not removed, it may stimulate ingrowth of fibroblasts and o Bacteria that produce this localized suppuration are
blood vessels which will lead to scarring called pyogenic bacteria
 Conversion of fibrinous exudate to scar tissue (organization)  Common example: acute appendicitis
within the pericardial sac leads to opaque fibrous thickening of  ABSCESSES are localized collections of purulent inflammatory
the pericardium and epicardium in the area of exudation
tissue
o If fibrosis is extensive  obliteration of pericardial space o Caused by suppuration buried in tissue, organ, or
 Thick, sticky with high cell and fibrin contents; ↑ risk of scar confined space
tissue
o Produced by seeding of pyogenic bacteria into tissues
o They have a central region which contains necrotic
leukocytes and tissue cells
 There is usually a zone of preserved neutrophils
around this necrotic focus
 Outside this region there may be vascular
dilatation and parenchymal and fibroblastic
proliferation
o May become walled-off and ultimately replaced by
connective tissue

Multiple fibrous adhesions (arrows) between loops of small intestine resulting from
previous abdominal inflammation

Multiple bacterial abscesses (arrows) in the lung in a case of bronchopneumonia.

Fibrinous pericarditis; deposits of fibrin on the pericardium

Abscess with neutrophils and cellular debris, surrounded by congested blood vessels

ULCERS
Morphologic feature: Fibrinous exudate  A local defect or excavation of the surface of an organ or
- Involves two surfaces in close proximity tissue that is produced by sloughing of inflamed necrotic
- Type of inflammation which often heals by ingrowth of fibrous tissue
tissue, which binds the adjacent surfaces together by means of  Can only occur when tissue necrosis and resultant
fibrous bands called adhesions inflammation exist on or near a surface

Page 20 of 44
TRANSCRIBERS: Bancod, Catindig, Magaoay, Ordoña
Acute & Chronic Inflammation & Tissue Repair

 Most common in:


o Mucosa of mouth, stomach, intestines, genitourinary tract
o Skin and subcutaneous tissue of lower extremities in
older people with CV distrubances
 Best exemplified by peptic ulcer of the stomach/duodenum
o Acute and chronic inflammation may coexist
o With chronicity, margins and base of ulcer develop
fibroblastic proliferation, scarring, accumulation of
lymphocytes, macrophages, plasma cells

Acute infection (cellulitis) of the finger


CHRONIC INFLAMMATION
 Response of prolonged duration (weeks or months)
 Inflammation, tissue injury and attempts at repair coexist in
varying combinations
 May follow acute inflammation or may begin insidiously as a
low-grade, smoldering response without any manifestations of
an acute reaction
 Characterized by a combination of leukocyte infiltration, tissue
damage, and fibrosis

CAUSES OF CHRONIC INFLAMMATION


1. Persistent infections by microorganisms that are difficult
to eradicate
o Often evoke an immune reaction called delayed-type
hypersensitivity
o Inflammatory response sometimes takes a specific pattern
called a granulomatous reaction

2. Hypersensitivity diseases
o Excessive and inappropriate activation of the immune
system
The morphology of an ulcer. A, A chronic duodenal ulcer. B, Low-power cross-section o In some conditions immune reactions develop against the
view of a duodenal ulcer crater with an acute inflammatory exudate in the base. individual’s own tissues, leading to autoimmune diseases
INFECTION o Auto-antigens evoke a self-perpetuating immune reaction
*From 2A 2017 trans that results in chronic tissue damage and inflammation
 Rheumatoid arthritis, multiple sclerosis
- Denote an inflammatory process caused by disease-producing
o In other cases it is the result of unregulated immune
organisms
responses against microbes
- Ending –it is is appended to the name of the tissue or organ to
o Immune responses against common environmental
indicate an inflammatory process
substances are the cause of allergic diseases
- Ex. Appendicitis (appendix), hepatitis (liver), colitis(colon),
 Bronchial asthma
pneumonitis(lung)
o Autoimmune and allergic reactions are inappropriately
o Cellulitis – acute infection of the skin and deeper
triggered against antigens that are normally harmless
tissues
 The reactions serve no useful purpose and only
cause disease

3. Prolonged exposure to potentially toxic agents,


exogenous or endogenous
o An example is silica, when inhaled for prolonged periods,
results in an inflammatory lung disease called silicosis
o Atherosclerosis is thought to be a chronic inflammatory
process of the arterial wall induced by excessive
production and tissue deposition of endogenous
cholesterol and other lipids

MORPHOLOGIC FEATURES
Acute appendicitis. Marked inflammatory exudate on the surface of the appendix. 1. Infiltration with mononuclear cells which include
macrophages, lymphocytes and plasma cells
2. Tissue destruction induced by the persistent offending agent
or by inflammatory cells

Page 21 of 44
TRANSCRIBERS: Bancod, Catindig, Magaoay, Ordoña
Acute & Chronic Inflammation & Tissue Repair

3. Attempts at healing by connective tissue replacement of o Because of these activities, tissue destruction is one of
damaged tissue, accomplished by angiogenesis and fibrosis the hallmarks of chronic inflammation
 In short lived inflammation (irritant is eliminated), macrophages
ROLE OF MACROPHAGES eventually disappear.
- The dominant cellular player in chronic inflammation  In chronic inflammation, macrophage accumulation persists as a
- Secrete cytokines and growth factors that act on various cells result of continuous recruitment from the circulation and local
by: proliferation at the site of inflammation.
o Destroying foreign invaders and tissues
o Activating other cells (T lymphocytes) MACROPHAGE ACTIVATION
- Professional phagocytes that act as filters for particulate  In response to most stimuli, the first activation pathway is the
matter, microbes, and senescent cells classical one, designed to destroy offending agents
- Also function as effector cells that eliminate microbes in cellular
o Followed by alternative activation, which initiates tissue
and humoral immune responses
- Derived from: repair
o Hematopoietic stem cells in the bone marrow
o Progenitors in the embryonic yolk sac and fetal liver CLASSICAL MACROPHAGE ACTIVATION (M1)
during early development  May be induced by:
 Circulating cells of this lineage are called o Microbial products (endotoxin) which engage TLRs and
monocytes other sensors
- Member of the mononuclear phagocyte system o T-cell derived signals
o Old inaccurate name: reticuloendothelial system o Immune responses
o This system consists of closely related cells of bone o Foreign substances including crystals and particulate
marrow origin matter
o Includes blood monocytes and tissue macrophages o Classically activated macrophages (M1)
o Scattered in connective tissue or organs as: o Produce NO and ROS
 Kupffer cells (Liver) o Upregulate lysosomal enzymes
 Sinus histiocytes (spleen and lymph nodes) o These enhance their ability to kill ingested organisms and
 Alveolar macrophages (lungs) secrete cytokines that stimulate inflammation
 Microglia (CNS) o Important in host defense against microbes and in many
o Arise from a common precursor in the bone marrow inflammatory reactions
which then gives rise to blood monocytes o Also capable of injuring normal tissues
o Monocytes then migrate to tissues and differentiate into
macrophages ALTERNATIVE MACROPHAGE ACTIVATION (M2)
 Half-life of monocytes is about 1 day
 Lifespan of tissue macrophages is several Induced by cytokines other than IFN-gamma (IL-4 and IL-3
months or years produced by T lymphocytes and other cells
- Extravasation of monocytes is governed by the same factors  Not actively microbicidal and the cytokines may actually inhibit the
involved in neutrophil emigration classical activation pathway
o Involves adhesion molecules and chemical mediators Principal function of M2 is in tissue repair
with chemotactic and activating properties  Secrete growth factors that:
o Monocytes begin to emigrate into extravasculat tissue o Promote angiogenesis
quite early o Activate fibroblasts
 They may constitute predominant cell type within o Stimulate collagen synthesis
48 hours

 The impressive arsenal of mediators of these macrophages MACROPHAGE FUNCTIONS


makes them powerful allies in the body’s defense against  Products of activated macrophages eliminate injurious agents
unwanted invaders, but some weaponry can also induce tissue (microbes) and initiate process of repair
destruction when inappropriately activated.  Also responsible for much tissue injury in chronic inflammation
1. Ingestion and elimination of microbes and dead tissues
Page 22 of 44
TRANSCRIBERS: Bancod, Catindig, Magaoay, Ordoña
Acute & Chronic Inflammation & Tissue Repair

2. Initiation of the process of tissue repair, scar formation, o Produce antibodies directed against persistent foreign or
fibrosis self antigens or against altered tissue components
3. Secretion of mediators of inflammation (central to the
initiation and propagation of inflammatory reactions - In some chronic inflammatory reactions, the accumulated
a. Cytokines (TNF, IL-1, chemokines, etc) lymphocytes, antigen-presenting cells, and plasma cells cluster
b. Eicosanoids together to form lymphoid tissues resembling lymph nodes
4. Antigen presentatin to T lymphocytes and respond to o Called tertiary lymphoid organs
signals from T cells o This type of lymphoid organogenesis is often seen in the
a. Sets up a feedback loop that is essential for synovium of patients with long-standing rheumatoid
defense against many microbes by cell- arthritis and in the thyroid in Hashimoto’s thyroiditis
mediated immune responses

ROLE OF LYMPHOCYTES
 Microbes and other environmental antigens activate T and B
lymphocytes, amplifying and propagating chronic inflammation
 Major function is as the mediators of adaptive immunity
 Often present in chronic inflammation and when activated,
inflammation tends to be persistent and severe
 May be the dominant population in the chronic inflammation
seen in autoimmune and other hypersensitivity diseases

1. T LYMPHOCYTES
-
o Mobilized in both Ab-mediated and cell-mediated immune OTHER CELLS IN CHRONIC INFLAMMATION
reactions EOSINOPHILS
o Ag-stimulated lymphocytes use adhesion molecule pairs
o Abundant in reactions mediated by IgE and in parasitic
and chemokines to migrate to site of inflammation
infections
o Cytokines, mainly TNF and IL-1, and chemokines promote
o Recruitment is driven by adhesion molecules similar to
leukocyte recruitment
those used ny neutrophils
o Bidirectional interaction between macrophages and
o Eotaxin
lymphocytes:
 A chemokine important in eosinophil recruitment
o Eosinophil granules contain major basic protein
Macrophages display antigens to T cells and produce cytokines  Highly cationic protein that is toxic to parasites but
(IL-12) that stimulate T-cell responses also lyses mammalian epithelial cells

Activated T lymphocytes produce cytokines, especially IFN-γ, MAST CELLS
which is a powerful activator of macrophages o Expresses the receptor FcεRI that binds to the Fc portion
↓ of IgE Abs
Interaction between T cell and macrophages causes the reaction o The IgEs on the mast cell recognize antigens
to be chronic and severe  This causes degranulation and release of mediators
such as histamine and prostaglandin
 Occurs during allergic reactions to food, venom, drugs
o CD4+ T lymphocytes
 Promote inflammation and influence the nature of the NEUTROPHILS
inflammatory reaction
 Although characteristic of acute inflammation, they are
 Greatly amplify the early inflammatory reaction that is
also present in chronic inflammation
induced by recognition of microbes and dead cells
 Induced either by persistent microbes or by mediators
 3 subsets:
produced by activated macrophages and T lymphocytes
 TH1  produce cytokine IFN-gamma, activating
 In chronic bacterial infection of bone (osteomyelitis), a
macrophages via classical pathway
neutrophilic exudate can persist for many months
 TH2  secretes IL-4, IL-5, IL-13, which recruit and  Also important in chronic damage induced in lungs by
activate eosinophils; also responsible for the smoking
alternative pathway of macrophage activation  Pattern of inflammation is called acute on chronic
 TH17  secrete TH17 and other cytokines,
inducing secretion of chemokines responsible for GRANULOMATOUS INFLAMMATION
recruiting neutrophils and monocytes into the
 A distinctive pattern of chronic inflammation
reaction
 Both TH1 and TH17 are involved in defense against  Characterized by collections of activated macrophages, often
autoimmune diseases with T lymphocytes, sometimes with central necrosis
 TH2 cells are important in defense against helminthic  Granuloma
parasites and in allergic inflammation o A cellular attempt to contain an offending agent that is
difficult to eradicate
o There is often a strong activation of T cells leading to
2. B LYMPHOCYTES
macrophage activation, which can cause injury to normal
o Antibody-producing plasma cells are from activated B
tissues
lymphocytes

Page 23 of 44
TRANSCRIBERS: Bancod, Catindig, Magaoay, Ordoña
Acute & Chronic Inflammation & Tissue Repair

o Activated macrophages develop abundant cytoplasm and 1. FEVER


begin to resemble epithelial cells, thus called epithelioid o Elevation of body temperature usually by 1 to 4 degrees
cells Celsius
 Some may fuse, forming multinucleate giant o Produced in response to pyrogens that act by stimulating
cells prostaglandin synthesis in the vascular and perivascular
o May also develop in some immune-mediated diseases cells of the hypothalamus
(Crohn’s disease and sarcoidosis) o Bacterial endotoxins (exogenous pyrogens) stimulates
 In persistent T cell responses to certain microbes, T cell- leukocytes to release IL-1 and TNFα (endogenous
derived cytokines are responsible for granuloma formation pyrogens) that increase enzymes (COXs) that convert AA
 Tuberculosis is the prototype of a granulomatous disease to prostaglandins
caused by infection o In the hypothalamus, PGE2 stimulates neurotransmitter
production (cAMP) which resets the temperature set point
TWO TYPES OF GRANULOMAS to a higher level
1. Foreign body granulomas o NSAIDS reduce fever by inhibiting prostaglandin
 Incited by relatively inert foreign bodies in the absence synthesis
of T cell-mediated immune responses 2. ACUTE-PHASE PROTEINS
 Form around materials such as talc, sutures or other o Mostly synthesized in the liver
fibers are enough to preclude phagocytosis by a o Three best known acute phase proteins:
single macrophage and do not incite any specific  C-reactive protein (CRP)
inflammatory or immune response  Serum amyloid A protein (SSA)
 Fibrinogen
 Epithelioid and giant cells are apposed to the surface
o Synthesis of these proteins is up-regulated by IL-6 (CRP
of the foreign body
and fibrinogen) and IL-1 or TNF (for SSA)
2. Immune granulomas
o CRP and SSA act as opsonins and fix complement
 Caused by agents that are capable of inducing a T
 Also bind chromatins, aiding in clearing necrotic
cell-mediated immune response
cell nuclei
 Produces granulomas when inciting agent is difficult to o Fibrinogen binds to RBCs and causes them to form stacks
eradicate (rouleaux) to sediment more rapidly (thus the basis for
 Macrophage activates T cells to produce cytokines measuring ESR during inflammation)
(IL-2 and IFN-gamma), activating other T cells  o Prolonged production during chronic inflammation causes
response perpetuation secondary amyloidosis
 Mycobacterium tuberculosis infection forms a o Hepcidin is an iron regulator whose production is
granuloma called a tubercle increased during acute-phase response
 Characterized by the presence of a central  Increased levels reduce availability of iron
caseous necrosis  Responsible for anemia associated with chronic
inflammation
3. LEUKOCYTOSIS
o Especially involved in those induced by bacterial infectiors
o Leukocyte counts increase to 15,000 to 100,000 cells/uL
o These elevations are referred to as leukemoid reactions
o Leukocytosis occurs because of accelerated release of
cells from the bone marrow postmitotic reserve pool due to
cytokines including TND and IL-1
 There is therefore a rise in more immature cells in
blood (shift to the left)
o Prolonged infection also induces proliferation of precursors
in the bone marrow due to increased colony-stimulating
factor production
 To compensate for the loss of these cells in the
inflammatory reaction
o Bacterial infections  neutrophilia
o Viral infections  lymphocytosis
o Allergies, parasitic infections  eosinophilia
o Typhoid fever, viral infections, rickettsiae, protozoa 
leukopenia
4. OTHERS
o Increased pulse and BP
o Decreased sweating, because of redirection of blood flow
SYSTEMIC EFFECTS OF INFLAMMATION from ctaneous to deep to minimize heat loss
 Systemic changes associated with acute inflammation are o Rigors
collectively called the ACUTE-PHASE RESPONSE or the o Chills
systemic inflammatory response syndrome (SIRS) o Anorexia
 IL-1, IL-6, TNF are important mediators of the acute-phase o Somnolence
reaction o Malaise, because of actions of cytokines on brain cells
 The acute-phase response consists of:

Page 24 of 44
TRANSCRIBERS: Bancod, Catindig, Magaoay, Ordoña
Acute & Chronic Inflammation & Tissue Repair

o In sepsis, large amounts of bacteria and their producs I. Tissue Proliferative Activity
stimulate production of enourmous quantities of several  Three groups of the body tissues according to proliferative
cytokines (TNF, IL-1) activity of the cells
o High blood levels of cytokines (clinical triad of septic 1) Continuously dividing (labile) tissues
shock)
 Disseminated intravascular coagulation o proliferate throughout life
 Hypotensive shock o replaces those that are destroyed
 Metabolic disturbances (insulin resistance, o skin epithelia, oral cavity, vagina, cervix, lining mucosa of
hyperglycemia) excretory ducts of glands of the body, columnar
epithelium of GI and uterus, transitional epithelium of
TISSUE RENEWAL, REGENERATION AND REPAIR urinary tract, cells of bone marrow, and hematopoietic
REGENERATION tissues
 Complete restitution of lost or damaged tissue o mature cells are derived from stem cells
 proliferation of lost or damaged tissues to replace lost structures 2) Quiescent tissue
 rarely seen in whole organs and complex tissues of mammals o low level of replication
o Liver growth after partial resection or necrosis consists of o can undergo rapid division when stimulated, thus, are
compensatory growth rather than true regeneration capable of reconstituting the tissue of origin
o tissues with high proliferative capacity (e.g. hematopoietic o parenchymal cells of the liver, kidney and pancreas,
system and epithelia of skin and GI tract) can renew mesenchymal cells such as fibroblast and smooth
themselves continuously and can regenerate after injury as muscles, vascular endothelial cells, lymphocytes and
long as stem cells are not destroyed other leukocytes, chondrocytes, and osteocytes
3) Nondividing tissues
REPAIR o cannot undergo mitotic division in postnatal life
 restores some original structures but can cause structural o neurons, skeletal and cardiac muscle cells
derangements
 most often consists of a combination of regeneration and II. Stem Cells
scar formation by deposition of collagen  characterized by their self-renewal properties and by
 scar formation is the predominant healing process when their capacity to generate differentiated cell lineages
the ECM framework is damaged by severe injury  maintained during the life of the organism by two
 chronic inflammation that accompanies persistent injury mechanisms: 1) obligatory asymmetric replication, in
also stimulates scar formation because of the local which in every cell division, one daughter cell retains its
production of cytokines and growth factors that promote self-renewing capacity while the other proceeds to a
fibroblast proliferation and collagen synthesis differentiation pathway, and 2) stochastic
Fibrosis - extensive deposition of collagen differentiation, in which a stem cell population is
Extracellular matrix maintained by the balance between stem cell divisions
 essential for wound healing that generate either two self-renewing stem cells or two
o provide framework for cell migration cells that will differentiate
o maintains correct cell polarity for the reassembly of Adult stem cells
multilayer structures  more restricted capacity to generate different cell types
o participate in angiogenesis compared to embryonic stem cells
 cells in the ECM (fibroblasts, macrophages, etc.) produce  have been identified in many tissues (skin, lining of gut, cornea
cytokines, growth factors, chemokines that are critical for and particularly in hematopoietic tissue)
regeneration and repair  reside in special microenviroment called niches composed of
Control of Normal Cell Proliferation and Tissue Growth mesenchymal, endothelial and other cell types
Determinants of cell population: o niche cells generate or transmit stimuli that
1. rate of cell proliferation regulate stem cell-renewal and generation of
2. differentiation progeny cells
3. death by apoptosis  Research has now demonstrated that reprogramming of
 cell proliferation can be stimulated by physiologic and differentiated cells in humans and mice into pluripotent stem
pathologic conditions and is largely controlled by signals from cells (cells that can generate all tissues of the body) by
the microenvironment that can either stimulate of inhibit transduction of genes encoding ES cell transcription factors.
proliferation These cells are called induced pluripotent stem cells (iPS cells)
o proliferation of endometrial cell under estrogen
stimulation and TSH-mediated replication of thyroid cells A. Embryonic stem cells
(physiologic proliferation)  located in inner cell mass of blastocysts in early embryonic
o physiologic stimulation can be pathologic when excessive development
(i.e. nodular prostatic hyperplasia resulting from  can be maintained in culture as undifferentiated cell lines or be
dihydrotestoterone stimulation) induced to differentiate into specific lineages such as heart and
liver cells
 may be in the future be used to repopulate damaged organs

Page 25 of 44
TRANSCRIBERS: Bancod, Catindig, Magaoay, Ordoña
Acute & Chronic Inflammation & Tissue Repair

B. Reprogramming of Differentiated Cells: Induced of adult brains: subventricular zone (SVZ) and dentate gyrus of
Pluripotent Stem Cells the hippocampus
 can be done by transferring nucleus of differentiated adult cells  It is not clear what might be the purpose of adult neurogenesis
to an enucleated oocyte Skin
 These cells can be transplanted to the patient to repopulate  Stem cells are located in three different areas of the epidermis:
damaged organs hair follicle bulge, interfollicular areas of the surface epidermis,
 therapeutic cloning as well as reproductive cloning are inefficient and sebaceous glands.
and often inaccurate  The bulge areas of the hair follicle constitutes a niche for stem
o due to deficiency in histone methylation in reprogrammed cells that produce all of the cell lineages of the hair follicle.
ES cells, which results in improper gene expression  Interfollicular cells are scattered individually in the epidermis and
C. Adult (Somatic) Stem Cells are not contained in niches. They divide infrequently but
 present in tissues that continually divide such as the bone generate transit amplifying cells that generate the differentiated
marrow, the skin, and the lining of the GI tract epidermis.
 may also be present in organs such as liver, pancreas, and Intestinal epithelium
adipose tissue  In the small intestine, crypts are monoclonal structures derived
 generate rapidly dividing cells known as transit amplifying cells from single stem cells: the villus is a single differentiated
o these cells lost capacity for self-perpetuation compartment that contains cells from multiple crypts
and give rise to cells with restricted development  Stem cells may be located above Paneth cells in the small
potential known as progenitor cells intestine, or at the base of the crypt, as in the case in the colon.
Transdifferentiation Skeletal and Cardiac Muscle
 change in differentiation of a cell from one type to another  Skeletal muscle myocytes do not divide, even after injury
Developmental plasticity  growth and regeneration of injures skeletal myocytes occur by
 capacity of a cell to transdifferentiate into diverse lineages replication of satellite cells
 Hemopoietic stem cells (HSC) had been shown to  They are located beneath the myocyte basal lamina, constitute a
transdifferentiate to other cell types, such as neurons, skeletal reserve pool of stem cells that can generate differentiated
and cardiac myocytes and hepatocytes myocytes after injury
D. Stem Cells in Tissue Homeostasis Cornea
Bone Marrow  Transparency of the cornea depends on the integrity of the
 contains HSCs and stromal cells (also known as multipotent outermost corneal epithelium, which is maintained by limbal
stromal cells, mesenchymal cells or MSCs). stem cells (LSCs)
o Hematopoietic stem cells - generate all blood cell  These cells are located at the junction between the epithelium of
lineages, can reconstitute bone marrow after depletion the cornea and conjunctiva
caused by disease or irradiation, widely used for the  Hereditary or acquired conditions that result in LSC deficiency
treatment of hematologic diseases. They can be collected and corneal opacification can be treated by limbal
directly from bone marrow, from umbilical cord blood, and transplantation or LSC grafting.
from the peripheral blood of individuals receiving
cytokines such as granulocyte-macrophage colony- Cell Cycle and the Regulation of Cell Replication
stimulating factor. The human bone marrow produces  Cell proliferation is a tightly regulated process that involves large
6
around 1.5 x 10 blood cells per second. number of molecules and interrelated pathways
o Marrow stromal cells - Multipotent cells that have
potentially therapeutic applications, because they can Salient Features of the Process of Cellular Proliferation
generate chondrocytes, osteoblasts, adipocytes,  replication of cells is stimulated by growth factors or by
myoblasts, and endothelial cell precursors depending on signalling from ECM components through integrins
which tissue they migrate. They migrate to injured tissues  The cell undergoes the cell cycle in order to achieve DNA
and generate stromal cells and other cell lineages, but do replication and division
not seem to participate in normal tissue homeostasis.  Phases:
Liver o G1 (presynthetic)
 contains stem cells/progenitor cells in the canals of Hering o S (DNA synthesis)
 cells located in this niche can give rise to a population of o G2 (premitotic)
precursor cells known as oval cells which are bipotential o M (mitotic)
progenitors, capable of differentiating into hepatocytes and o G0 (for quiescent cells)
biliary cells  Each phase is dependent on the proper activation and
 oval cell proliferation and differentiation are prominent among completion of the previous one
patients recovering from fulminant heaptic failure, liver  cycle stops at a place at which an essential gene function is
tumorigenesis and in some cases of chronic hepatitis and deficient
advanced liver cirrhosis  has multiple control and redundancies, particularly during
Brain transition between G1 and S phases.
 Neural precursor cells (NPC), capable of generating neurons,  Controls include activators and inhibitors, as well as sensors
astrocytes, oligodendrocytes, have been identified in two areas responsible for checkpoints.
 Cells can enter G1 either from G0 or after completing mitosis
Page 26 of 44
TRANSCRIBERS: Bancod, Catindig, Magaoay, Ordoña
Acute & Chronic Inflammation & Tissue Repair

 This transition involves transcriptional activation of a large set of Epidermal Growth Factor (EGF) and Transforming Growth
genes, including proto-oncogenes and genes required for Factor α (TGF-α)
ribosome synthesis and protein translation  Belong to the EGF family
 Cells in G1 progress through the cycle to reach a critical stage at  Share a common receptor (EGFR)
the G1/S transition, known as the restriction point, a rate-limiting  EGF is mitogenic for a variety of epithelial cells, hepatocytes,
step for replication. Upon passing this restriction point, normal and fibroblasts
cells become irreversibly committed to DNA replication.  widely distributed in tissue secretions and fluids
 Progression through the cell cycle, particularly at the G1/S  in healing skin wounds, EGF is produced by keratinocytes,
transition, is tightly regulated by cyclins and associated enzymes macrophages and other inflammatory cells that migrate into the
called cyclin-dependent kinases (CDKs) area
 CDKs acquire catalytic activity by binding to and forming  TGF-α is involved in epithelial cell proliferation in embryos and
complexes with the cyclins adults, and in malignant transformation on normal cells to
 Activated CDKs drive the cell cycle by phosphorylating proteins cancer
that are critical for cell cycle transition  shares most of the biologic activities of EGF
 The activity of cyclin-CDK complexes is tightly regulated by CDK Hepatocyte Growth Factor (HGF)
inhibitors. Some growth factors shut off production of these  originally isolated from platelets and serum
inhibitors. Embedded in the cell cycle are surveillance
 identical to previously identified GF from fibroblasts known as
mechanisms that are geared primarily by sensing damage to scatter factor
DNA and chromosomes. These are called checkpoints.
 has mitogen effects on hepatocytes and most epithelial cells,
 G1/S checkpoint monitors intergrity of DNA before replication, including cells from biliary epithelium, and epithelial cells of the
whereas G2/M checkpoint checks DNA after replication and lungs, kidney, mammary gland, and skin.
monitors whether the cell can safely enter mitosis.
 acts as a morphogen in embryonic development, promotes cell
 When cells sense DNA damage, checkpoint activation delays scattering and migration, and enhances survival of hepatocytes.
cell cycle and triggers DNA repair mechanisms.
 produced by fibroblasts and most mesenchymal cells,
 If DNA damage is to severe, the cell is eliminated by apoptosis endothelial cells and liver nonparenchymal cells.
or enter a nonreplicative phase called senescence, primarily
 Its receptor, c-MET, is often highly expressed or mutated in
through p53-dependent mechanisms.
human tumors, especially in renal and thyroid papillary
 Checkpoint defects that allow cells with DNA strand breaks and carcinomas.
chromosome abnormalities to divide produce
 HGF signalling is required for survival during embryonic
development
GROWTH FACTORS
Platelet Derived Growth Factor (PDGF)
 proliferation of many cell types is driven by polypeptides known  all isoforms exert their effects by binfing to two cell surface
as growth factors receptors, designated as PDGF α and β, which have different
 may also promote cell survival, locomotion, contractility, lignad specificities.
angiogenesis, and differentiation that may be as important as  PDGF is stored in platelet granules and is released on platelet
their growth-promoting effects activation
 all GFs function as ligands that bind to specific receptors, which  produced by activated macrophages, endothelial cell, smooth
deliver signals to the target cells muscle, and many tumor cells
 These signals stimulate the transcription of genes that may be  causes migration and proliferation of fibroblasts, smooth muscle
silent in resting cells, including genes that control cell cycle and cells and monocytes to areas of inflammation and healing skin
progression. wounds
FROM RUBIN’S: Vascular Endothelial Growth Factor (VEGF)
 potent inducer of blood vessel formation in early development
(vasculogenesis)
 has a central role in growth of new blood vessels (angiogenesis)
 promotes angiogenesis in chronic inflammation, wound healing,
and tumors
 signal through tyrosine kinase receptors
 VEGFR-2 is the main receptor for vasculogenic and angiogenic
effects of VEGF
 VEGFR-1 may facilitate in the mobilization of endothelial stem
cells and has a role in inflammation
 VEGFR-3 is involved in production of lymphatic vessels
(lymphangiogenesis)
Fibroblast Growth Factor (FGF)
 family of growth factors containing more than 20 members
 transduce signals through tyrosine kinase receptors
 FGF-7 is referred to as keratinocyte growth factor (KGF)

Page 27 of 44
TRANSCRIBERS: Bancod, Catindig, Magaoay, Ordoña
Acute & Chronic Inflammation & Tissue Repair

 contribute to wound healing responses, hematopoiesis,  Paracrine Signaling


angiogenesis, development and other several processes o one cell type produces the ligand, which then acts on
through several functions: adjacent target cells that express the appropriate
o FGF-2 nad KGF contribute to reepithelialization of receptor
skin wounds o common in connective tissue repair of healing wound
o FGF-2 has the ability to induce angiogenesis o also necessary for hepatocyte replication during liver
o FGFs have been implicated in the differentiation of regeneration
specific lineages of blood cells and development of o in Notch effects in embryonic development, wound
bone marrow stroma healing, and renewing tissues
o FGFs play a role in skeletal and cardiac muscle  Endocrine signaling
development, lung maturation, and the specification o hormones synthesized by cells of endocrine organs
of the liver from endodermal cells act on target cells distant from their site of synthesis ,
Transforming Growth Factor β (TGF-β) and Related Growth being usually carried by the blood.
Factors A. Receptors and Signal Transduction Pathways
 TGF-β belongs to a superfamily of about 30 members 1. Intrinsic Tyrosine Kinase
 homodimeric protein produced by a variety of different cell  ligands include most GFs such as EGF, TGF, HGF, PDGF,
types, including platelets, endothelial cells, lymphocytes, and VEGF, FGF, c-KIT ligand, and insulin.
macrophages  binding of ligand → dimerization of receptor → tyrosine
 binds to two cell surface receptors (types I and II) with phosphorylation → and activation of receptor tyrosine kinase →
serine/threonine kinase activity and triggers the phosphorylation active kinase phosphorylates and activates many downstream
of cytoplasmic transcription factors called Smads effector molecules.
 Phosphyrylated Smads form heterodimer with Smad 4 2. Receptors lacking intrinsic tyrosine kinase activity that
 heterodimer enters nucleus and associate with other DNA recruit kinases
binding proteins to activatr or inhibit gene transciption  ligands include many cytokines such as IL-2, IL-3 and other
 TGF-β has multiple and often opposing effects depending on the interleukins; INF-α,β,γ; erythropoietin; granulocyte colony
tissue and the type of injury stimulating factor; growth hormone and prolactin
 Agents that have multiple effects are called pleiotropic  ligand binds to receptor → activates JAK to transmit
 because of the large diversity of TGF-β effects, it has been said extracellular signal to the nucleus → JAKs link the receptor with
that TGF-β is pleiotropic with a vengeance and activate cytoplasmic transcription factors called STATs →
o TGF-β is the growth inhibitor for most epithelial cells. It shuttle to nucleus to activate transcription
blocks the cell cycle by increasing the expression of cell 3. G protein-coupled receptors
cycle inhibitors  transmit signal into the cell through trimeric GTP binding protein
o TGF-β is a potent fibrinogenic agent that stimulates (G proteins)
fibroblast chemotaxis and enhances production of  contain seven transmembrane alpha helices
collagen, fibronectin and proteoglycans. It is involved in  constitute the largest family of plasma membrane receptors
fibrosis in a variety of chronic inflammatory conditions  ligands include chemokines, vasopressin, serotonin, histamine,
particularly in lungs, kidney and liver. High TGF epinephrine and norepinephrine, calcitonin, glucagon, PTH,
expression also occurs in hypertrophic scars, systemic corticotropin and rhodopsin
sclerosis and Marfan syndrome.  enermous number pf pharmaceutical drugs target such
o TGF-β has a strong anti-inflammatory effect but may receptors
enhance some immune functions. In mice, lack in TGF-β
 binding of ligand → change in conformation of receptor →
induces defects in regulatory T cell leading to widespread activation and allowing interaction with many different G proteins
inflammation with abundant T-cell proliferation and CD4+ → exchange of GDP to GTP to activate the protein
differentiation into TH1 and TH2 helper cells
 among the many branches of the signal transduction pathway
Cytokines
involves the cAMP as second messengers
 have important functions as mediators of inflammation and  can also produce IP3
immune responses.
 IP3 → increase intracellular calcium → target cytoskeletal
 Can also be considered as growth factors, because they have proteins, chloride- and potassium-activated ion pumps →
growth promoting activities for a variety of cells 2+
enzymes such as caplain, and Ca binding proteins such as
 TNF and IL-1 participate in wound healing reactions Calmodulin
 TNF and IL-6 are involved in the initiation of liver regeneration 4. Steroid Hormone Receptors
 Generally located in the nucleus
SIGNALING MECHANISMS IN CELL GROWTH
 function as ligand dependent transcription factorligand diffuses
 Autocrine SIgnaling through cell membrane → bind inactive receptors, causing their
o cells respond to the signaling molecule that they activation → activated receptors bind to specific DNA sequence
themselves secrete, establishing an autocrine loop known as Hormone Response Element (HRE) within target
o plays a role in liver regeneration and the proliferation genes
of antigen-stimulated lymphocytes  also include thyroid hormone, vitamin D and retinoids
o Tumors usually stimulate their own proliferation  A group of receptors belonging to this family are called
though an autocrine loop peroxisome proliferator-activated receptors
Page 28 of 44
TRANSCRIBERS: Bancod, Catindig, Magaoay, Ordoña
Acute & Chronic Inflammation & Tissue Repair

o they are involved in a broad range of responses that  two major restriction points for hepatocyte replication: G 0/G1
include adipogenesis, inflammation and transition that bring quiescent hepatocytes into cell cycle and the
atherosclerosis G1/S transition
B. Transcription Factors  Gene expression in the regenerating liver proceeds in phases,
 modulates gene transcription starting with the immediate early gene response, which is a
 used by signal transducion systems of growth factors when transient response that corresponds to the G0/G1 transition.
information is transferred to the nucleus More than 70 genes are activated during this response,
 have a modular design and contain domains for DNA binding including proto-oncogenes c-FOS and c-JUN, whose products
and for transcriptional regulation. dimerize to form the transcription factor AP-1; c-MYC, which
 growth factors induce the synthesis and activity of transcription encodes a transcription factor that activates many different
factors genes; and other transcription factors such as NF-κB, STAT-3,
 cellular events requiring rapid responses do not rely on new and C/EBP.
synthesis on TFs but depend on post-translational modifications  The immediate early gene response sets the stage for the
that lead to their activation. sequential activation of multiple genes, as hepatocytes progress
 Modifications include: to the G1 phase.
o heterodimerization, as for the products of the proto-  quiescent hepatocytes become competent to enter the cell cycle
oncogenes c-FOS and c-JUN to form transcription factor through a priming phase that is mostly mediated by the
activator protein-1 (AP-1) which is activated by MAP cytokines TNF and IL-6, and the components of the complement
Kinase signaling pathways system.
o phosphorylation, as for JAK/STAT pathway  Priming signals activate several signal transduction pathway as
o release of inhibiion to permit migration into nucleus, as for a necessary prelude to cell proliferation.
NF-κB  Under the stimulation of HGF, TGFα, and HB-EGF, primed
o release from membranes by proteolytic cleavage, as for hepatocytes enter the cell cycle and undergo DNA replication
Notch Receptors.  Norepinephrine, serotonin, insulin, thyroid, and growth hormone,
Mechanisms of Tissue and Organ Regeneration act as adjuvants for liver regeneration, facilitating the entry of
 the capacity for regeneration of regeneration of whole tissues hepatocytes into the cell cycle
and organs have been lost in mammals  individual hepatocytes replicate once or twice during
 attributed to absence of blastema formation (the source of cells regeneration and then return to quiescence in a strictly regulated
for regeneration) and to the rapid fibroproliferative response sequence of events, but the mechanism for growth cessation
after wounding have not been established
 Wnt/β-catenin is a highly conserved pathway that participates in  Intrahepatic stem or progenitor cells do not play a compensatory
regeneration in other animals growth that occurs after partial hpatectomy
 In mammals, Wnt/β-catenin modulates stem cell functions in the  However, endothelial cells and other nonparenchymal cells in
intestinal epithelium, bone marrow, and muscle, participates in the regenerating liver may originate from bone marrow
liver regeneration after partial hepatectomy, and stimulates oval precursors
cell proliferation after liver injury
SIGNALING MECHANISMS IN CELL GROWTH
SIGNALING MECHANISMS IN CELL GROWTH  tissue repair and regeneration depend not only on the activity of
 Human liver has a remarkable capacity to regenerate, as soluble factors, but also on the interaction between cells and the
demonstrated by its growth after partial hepatectomy, which may components of the extracellular matrix (ECM).
be performed for tumor resection or for living-donor hepatic  ECM regulates the growth, proliferation, movement, and
transplantation differentiation of cells living within it
 In humans, resection of approximately 60% of the liver in living  ECM sequesters water to provide turgor to soft tissues and
donors results in the doubling of the liver remnant in about one minerals that give rigidity to bone
month  Various functions include
 Restoration of the liver mass is achieved without the regrowth of o mechanical support for cell anchorage and migration,
the lobes that were resected at the operation and maintenance of cell polarity
 Instead, growth occurs by enlargement of the lobes that remain o control of cell growth
after the operation, a process known as compensatory growth or o maintenance of cell differentiation
compensatory hyperplasia o Scaffolding for tissue renewal
 In both humans and rodents, the end point of liver regeneration  maintenance of normal tissue structure
after partial hepatectomy is the restitution of functional mass requires a basement membrane or stromal
rather than the reconstitution of the original form scaffold
 Hepatocyte proliferation in the regenerating liver is triggered by  integrity of basement membrane or stroma
the combined actions of cytokines and polypeptide growth of parenchymal cells is critical for organized
factors and is strictly dependent on paracrine effects of growth regeneration of tissues
factors and cytokines such as HGF and IL-6 produced by  restitution of injured tissues to normal
hepatic nonparenchymal cells. structure occurs only if ECM is not
damaged

Page 29 of 44
TRANSCRIBERS: Bancod, Catindig, Magaoay, Ordoña
Acute & Chronic Inflammation & Tissue Repair

 disruption of these structures leads to  inherited defects in fibrillin result in formation of abnormal
collagen deposition and scar formation elastic fibers in Marfan syndome, manifested by changes in
o Establishment of tissue microenvironments cardiovascular system (aortic dissection and the skeleton)
o Storage and presentation of regulatory molecules
 FGF and HGF are stored in the ECM of C. Cell Adhesion Proteins
some tissues  also called CAMs (cell adhesion molecules)
 allows rapid deployment often local injury or  classified into four main families: immunoglobulin family
during regeneration CAMs, cadherins, integrins and selectins
 ECM is composed of three groups of macromolecules:  function as transmembrane receptors but are sometimes
o fibrous structural proteins (collagen and elastin to stored in the cytoplasm
provide tensile strength and recoil)
 as receptors, CAMs can bind to similar or different molecules
o adhesive glycoproteins (connect the matrix elements
in other cells, providing for interaction between same cells
to one another and to cells)
(homotypic interaction) or different cell types (heterotypic
o proteoglycans and hyaluronan to that provide resilience and
interaction)
lubrication
 Integrins bind to ECM proteins such as fibronectin, laminin,
 These molecules assemble to form two basic forms of ECM:
and osteopontin providing a connection between cells and
Intestinal matrix and basement membranes
ECM, and also to adhesive proteins in other cells, establishing
o interstitial matrix is found in spaces between
cell-to-cell contact
epithelial, endothelial, and smooth muscle cells, as
 Fibronectin
well as in connective tissue
o large protein that binds to many molecules, such as
 consists mostly of fibrillar and nonfibrillar
collagen, fibrin, proteoglycans, and cell surface
collagen, elastin, fibronectin, proteglycans,
receptors
and hyaluronan.
o consists of two proteoglycan chain, held together by
o Basement membranes are closely associated with
disulfide bonds.
cell surfaces, and consists of nonfibrillar collagen
o fibronectin messenger RNA has two splice forms,
(mostly type IV), laminin, heparin sulfate and
giving rise to tissue fibronectin and plasma
proteoglycans.
fibronectin
o plasma form binds to fibrin, helping to stabilize blood
MAIN COMPONENTS OF ECM
clot that fills the gaps created by wounds, and serve
A. Collagen
as a stratum for ECM deposition and formation of
 most common protein in the animal world provisional matrix during wound healing
 provides extracellular framework for all multicellular organisms  Laminin
 composed of three chains that form a trimer in the shape of a o most abundant glycoprotein in the basement
triple helix membrane
 repeating sequence in which glcycine is in every third position, o has binding domains for both ECM and cell surface
and it contains the specialized amino acids 4-hydroxyproline and receptors
hydroxylysine. o in the basement membrane, polymers of laminin and
 Type IV collagens have long but uninterrupted triple-helical collagen type IV form tightly bound networks
domains and form sheets instead of fibrils; they are the main o can also mediate the attachment of cells to
component of basement membrane, together with laminin. connective tissue substrates
 genetic dects in collagen prduction cause many inherited  Cadherins and Integrins
syndromes, including various forms of the Ehlers-Danlos o link the cell surface with the cytoskeleton through
syndrome and osteogenesis imperfect binding of actin and intermediate filaments
o provides mechanism, especially integrins, for the
B. Elastin, Fibrillin and Elastic Fibers transmission of mechanical force and the activation
 tissues such as blood vessels, skin, uterus and lung require of intracellular signal transduction pathways that
elasticity for their function respond to these forces
 gives the ability of tissues to expand and recoil (collagen for o ligand binding to integrins causes clustering of the
tensile strength) receptors in cell membrane and formation of focal
adhesion complexes
 consists of a central core made of elastin, surrounded by a
peripheral network of microfibrils o cytoskeletal proteins that co-localize with integrins at
the cell focal adhesion complex include talin,
 substantial amounts of elastin are found in the walls of large
vincullin, and paxillin
blood vessels, such as aorta, uterus, skin, and ligaments
o integrin-cytoskeleton complexes function as
 fibrillin, which composes the microfibrillar network that
activated receptors and trigger a number of signal
surrounds the core, associates either with itself or with other
transduction pathways, including the MAP kinase,
components of ECM
PKC, and PI3K pathways, which are also activated
 microfibrils serve, in part, as scaffolding for deposition of
by growth factors
elastin and the assembly of elastic fibers
o integrins and GF receptors interact ("crosstalk") to
o also influence the availability of active TGFβ in the
transmit environmental signals to the cell that
ECM
regulates proliferation, apoptosis, and differentiation
Page 30 of 44
TRANSCRIBERS: Bancod, Catindig, Magaoay, Ordoña
Acute & Chronic Inflammation & Tissue Repair

o Cadherin, derived from "calcium-dependent lubrication to many types of connective tissue, notably for the
adherence protein" cartilage in joints.
o participates in interactions with cells of the same type  Hyaluronan concentration increases in inflammatory diseases
o these interactions connect the plasma membrane of such as rheumatoid arthritis, scleroderma, psoriasis, and
adjacent cells, forming two types of cell junctions osteoarthritis.
called 1) zonula adherens, small, spotlike junctions  Hyaluronidase fragments hyaluronan into lower molecular
located near the apical surface of epithelial cells, and weight molecules (LMW HA) that have different functions than
2) desmosomes, stronger and more extensive the parent molecule
junctions, present in epithelial and muscle cells  LMW HA produced by endothelial cells binds to CD44
o mIgration of keratinocytes in the re-epithelialization receptor on leukocytes, promoting the recruitment of
of skin wounds is dependent on the formation of leukocytes at the site of inflammation. In addition, they
desmosomal junctions stimulate the production of inflammatory cytokines and
o Linkage of cadherins with the cytoskeleton occurs chemokines by white cells recruited to the sites of injury.
through two classes of catenins
o β-catenin links cadherins with α-catenin, which, in HEALING BY REPAIR, SCAR FORMATION & FIBROSIS
turn, connects to actin, thus completing a connection  if tissure injury is severe or chronic and damages both
with the cytoskeleton. parenchymal cells and the stromal framework of the tissue,
o Cell to cell interactions mediated by cadherins and healing cannot be accomplished by regeneration.
catenins play a major role in regulating cell motility,
 In this event, the main healing process is repair by deposition
proliferation and differentiation and account for the
of collagen and other ECM components, causing the
inhibition of cell proliferation that occurs when
formation of scar
cultured normal cells contact each other ("contact
 repair is a fibroproliferative response that "patches" rather
inhibtion").
than restores the tissue
 Other Secreted Adhesion Molecules
 the term scar is used to describe replacement of parenchymal
o SPARC (secreted protein acidic and rich in cysteine),
cells in any tissue by collagen, as in the heart after MI.
also known as osteonectin, contributes to tissue
 Repair by connective tissue deposition includes the following
remodeling in response to injury and functions as
basic features:
angiogenesis inhibitor
o inflammation
o thrombospondins, also inhibits angiogenesis
o angiogenesis
o osteopontin (OPN) is a glycoprotein that regulates
o migration and proliferation of fibroblasts
calcification, is a mediator of leukocyte migration
o scar formation
involved in inflammation, vascular remodelling, and
o connective tissue remodeling
fibrosis in various organs
o tenascin family, involved in morphogenesis and cell  fibrosis
adhesion o excess deposition of connective tissue
o when inflammation becomes chronic due to
D. Glycosaminoglycans (GAGs) and Proteoglycans persistence of damage
 In most healing process, a combination of repair and
 make up the third type of component in the ECM, besides the regeneration occurs. The relative contribution of repair and
fibrous structural proteins and cell adhesion proteins regeneration are influenced by 1) the proliferative capacity of
 consists of long, repeating polymers of specific disaccharides the cells of the tissue, 2) integrity of the extracellular matrix;
 except hyaluronan, GAGs are linked to a core protein, forming and 3) the resolution of chronicity of the injury and
molecules called proteoglycans inflammation
 remarkable in their diversity I. Mechanisms of Angiogenesis
 ECM may contain several different core proteins, each  Blood vessels are assembled during embryonic development
containing different GAGs by vasculogenesis, in which a primitive vascular network is
 four structurally distinct family of GAGs: established from endothelial cell precursors (angioblasts) or
o heparan sulfate from dual hemopoietic/endothelial cell precursors called
o chondroitin/dermatan sulfate hemangioblasts
o keratan sulfate  blood vessel formation in adults, known as angiogenesis or
o hyaluronan (HA) neovascularization, involves the branching and extension of
 first three are synthesized in Golgi and ER adjacent, pre-existing vessels, but it can also occur by
as proteoglycans recruitment of endothelial progenitor cells (EPCs) from bone
 hyaluronan is produced at plasma marrow.
membrane by enzymes called hyaluronan Angiogenesis from preexisting vessels
synthases and is not linked to a protein
 there is vasodilation, increased permeability of the existing
backbone
vessels, degradation of ECM, and migration of endothelial
 Hyaluronan is abundant in heart valves, skin, skeletal tissues, cells.
synovial fluid, vitreous of the eye, and the umbilical cord.
 Major steps:
 Hyaluronan binds a large amount of water, forming a viscous o Vasodilation in response to NO, and VEGF-induced
hydrated gel that gives connective tissue the ability to resist increased permeability of the pre-existing vessel
compression forces. It also helps provide resilience and
Page 31 of 44
TRANSCRIBERS: Bancod, Catindig, Magaoay, Ordoña
Acute & Chronic Inflammation & Tissue Repair

o Proteolytic degradation of the basement membrane  Newly formed vessels are fragile and need to become
of the parent vessel by matrix metalloproteinases "stabilized". This requires the recruitment of pericytes and
(MMPs) and disruption of cell-to-cell contact between smooth muscle cells (periendothelial cells) and the deposition
endothelial cells by plasminogen activator of ECM proteins
o Migration of endothelial cells toward the angiogenic  Angioproteins 1 and 2 (Ang1 and Ang2), PDGF, and TGF-β
stimulus participate in the stabilization process
o Proliferation of endothelial cells, just behind the o Ang1 interacts with a receptor on endothelial cells
leading front of migrating cells. called Tie2 to recruit periendothelial cells
o maturation of endothelial cells, which includes o PDGF participates in the recruitment of smooth
inhibition of growth and remodeling into capillary muscle cells
tubes. o TGF-β stabilizes newly formed vessels by enhancing
o Recruitment of periendothelial cells (pericytes and production of ECM proteins.
vascular smooth muscle cells) to form mature o Ang1-Tie2 interaction mediates vessel maturation
vessels. from simple endothelial tubes into more elaborate
Angiogenesis of Endothelial Precursor Cells (EPCs) vascular structures and helps maintain endothelial
 can be recruited from the bone marrow into tissues to initiate qiescence.
angiogenesis o Ang2 also interacts with Tie2. It has the opposite
 EPCs may contribute to re-endothelialization of vascular effects: makes endothelial cells more responsive to
implants and the neovascularization of ischemic organs, stimuli or inhibition
cutaneous wounds, and tumors  Tissue hypoxia can influence physiologic and pathologic
 Number of circulating EPCs increases greatly in patients with angiogenesis
ischemic conditions  VEGF transcription is regulated by the transcription factor HIF,
A. Growth Factors and Receptors Involved in Angiogenesis which is induced by hypoxia.
 VEGF is the most common important GF in adult tissue B. ECM Proteins as Regulators of Angiogenesis
undergoing physiologic angiogenesis as well as angiogenesis  a key component of angiogenesis is the motility and directed
occuring in chronic inflammation, wound healing, tumors, and migration of endothelial cells, required for the formation of
diabetic retinopathy new blood vessels
 VEGFR-2, a tyrosine kinase receptor, is the most important in  These processes are controlled by several classes of
angiogenesis proteins, including 1) integrins, especially αVβ3, which is
 VEGF induces migration of EPCs in the bone marrow, and critical for the formation and maintenance of newly formed
enhances the proliferation and differentiation of these cells at blood vessels, 2) matricellular proteins, including
sites of angiogenesis thrombospondin 1, SPARC, and tenascin C, which destabilize
 VEGF signaling stimulates the survival of endothelial cells, cell-matrix interactions and therefore promote angiogenesis,
their proliferation, and their motility, initiating the sprouting of and 3) proteinases, such as plasminogen activators and
new capillaries MMPs, which are important in tissue remodeling during

Page 32 of 44
TRANSCRIBERS: Bancod, Catindig, Magaoay, Ordoña
Acute & Chronic Inflammation & Tissue Repair

endothelial invasion.
 Proteinases also cleave extracellular proteins, releasing
matrix-bound growth factors such as VEGF and FGF-2 that
stimulate angiogenesis
 Proteinases also release inhibitors such as endostatin, a small
fragment of collagen that inhibits endothelial proliferation and
angiogenesis.
II. Cutaneous Wound Healing
 Divided into three phases: inflammation, proliferation, and
maturation
 These phases overlap, and their separation is somewhat
arbitrary, but they help to understand the sequence of events
that take place in the healing of skin wounds.
 Initial injury causes platelet adhesion and aggregation, and
the formation of a clot on the surface of the wound, leading to
inflammation.
 In the proliferative phase, there is formation of granulation
tissue, proliferation and migration of connective tissue cells, Formation of Blood Clot
and re-epithelialization of wound surface.  wounding causes the rapid activation of coagulation
 Maturation involves ECM deposition, tissue remodeling, and pathways, which results in the formation of a blood clot on a
wound contraction wound surface.
Healing by primary union or by first intention  clot serves to stop bleeding and also as a scaffold for
o healing of a clean, uninfected surgical incision migrating cells, which are attracted by growth factors,
approximated by surgical sutures cytokines and chemokines released into the area.
o incision causes death of a limited number of  release of VEGF leads to increased vascular permeability and
epithelial and connective tissue cells and disruption edema
of epithelial basement membrane continuity
 dehydration occurs at the external surface of the clot, forming
o Re-epithelialization to close the wound occurs with
a scab that covers the wound.
formation of a relatively thin scar
 Within 24 hours, neutrophils appear at the margins of the
 The repair process is more complicated in excisional wounds
incision, and use the scaffold provided by the fibrin clot to
that create large defects on the skin surface, causing
march in. They release proteolytic enzymes to clean out
extensive loss of cells and tissue
debris and invading bacteria.
o healing of these wounds involves a more intense
Formation of Granulation Tissue
inflammatory reaction, the formation of abundant
granulation tissue, and extensive collagen  fibroblasts and endothelial cells proliferate in the first 24 to 72
hours of the repair process to form a specialized type of tissue
deposition, leading to the formation of a substantial
called granulation tissue, which is a hallmark of tissue repair
scar, which generally contracts
o This form of healing is referred to as healing by  characteristic histologic feature is the presence of new blood
secondary union or by second intention vessels (angiogenesis) and the proliferation of fibroblasts
 the new vessels are leaky, allowing the passage of plasma
fluid and proteins into the extracellular space. Thus, new
granulation tissue is often edematous.
 The amount of granulation tissue formed depends on the size
of tissue deficit created by the wound and the intensity of
inflammation
 Hence, it is much more prominent in healing by secondary
union
 by 5 to 7 days, granulation tissue fills the wound area and
neovascularization is maximal.
Cell Proliferation and Collagen Deposition
 Neutrophils are largely replaced by macrophages by 48 to 96
hours.
 Macrophages are key cellular constituents of tissue repair,
clearing extracellular debris, fibrin, and other foreign material
at the site of repair, and promoting angiogenesis and ECM
deposition
 Migration of fibroblasts to the site in injury is driven by
chemokines, TNF, PDGF, TGF-β, and FGF.
 Their subsequent proliferation is triggered by multiple growth
factors, including PDGF, EGF, TGF-β, FGF, and the
cytokines IL-1 and TNF.
Page 33 of 44
TRANSCRIBERS: Bancod, Catindig, Magaoay, Ordoña
Acute & Chronic Inflammation & Tissue Repair

 In 24 to 48 hours, spurs of endothelial cells move from the  Degradation of collagen and other ECM proteins is achieved by
wound edge (initially with little cell proliferation) along the cut matrix metalloproteinases (MMPs)
margins of the dermis, depositing basement membrane  MMPs include:
components as they move. o interstitial collagenase
 They fuse in the midline beneath the surface scab, producing o gelatinase
a thin, continuous epithelial layer that closes the wound. o stromelysins
 Concurrently with epithelialization, collagen fibrils become o family of membrane-bound MMPs
more abundant and begin to bridge the incision. At first, a  MMPs are produced by fibroblasts, macrophages, neutrophils,
provisional matrix containing fibrin, plasma fibronectin, and synovial cells, and some epithelial cells
type III collagen is formed, but this is replaced by a matrix  MMPs secretion is induced by GFs, cytokines and phagocytosis,
composed primarily of type I collagen. and is inhibited by TGF-β and steroids.
 TGF-β is the most important fibrinogenic agent Recovery of Tensile Strength
Scar Formation  Net collagen accumulation, however, depends not only on
 The leukocytic infiltrate, edema, and increased vascularity increased collagen synthesis but also decreased degradation
largely disappear during the second week.  usually at the end of the first week after removal of sutures from
 Blanching begins, accomplished by the increased incisional surgical wound, wound strength is approximately 10%
accumulation of collagen within the wound area and of unwounded skin
regression of vascular channels.  Wound strength increases rapidly over the next 4 weeks, slows
 Ultimately, the original granulation tissue scaffolding is down at approximately the third month after the original incision,
converted into a pale, avascular scar, composed of spindle- and reaches a plateau at about 70 - 80% of the tensile strength
shaped fibroblasts, dense collagen, fragments of elastic of unwounded skin
tissue, and other ECM components.  Lower tensile strength in the wounded area may persist for life
 By the end of the first month, the scar is made up of acellular III. Local and Systemic Factors that Influence Wound Healing
connective tissue devoid of inflammatory infiltrate, covered by  The adequacy of wound repair may be impaired by systemic
intact epidermis. and local host factors
Wound Contraction  Systemic factors include:
 Generally occurs on large surface wounds. o Nutrition (protein deficiency and vitamin C deficiency
 contraction decreases gap between dermal edges and by retards healing)
reducing the wound surface area o Metabolic status (diabetes mellitus delays healing)
 Hence, it is an important feature of healing by secondary o Circulatory status (inadequate blood supply or venous
union abnormalities that retard venous drainage, impairs
 initial steps of wound contraction involves the formation, at the healing)
edge of the wound, of a network of myofibroblasts that o hormones (glucocorticoids inhibit collagen synthesis due
express smooth muscle α-actin and vimentin. These cells to their anti-inflammatory effect)
have ultrastructural characteristics of smooth muscle cells,  Local Factors include:
contract in the wound tissue, and may produce large amounts o infection (delays healing due to persistence of injury)
of ECM components. o Mechanical factors (early motion of healing
 Myofibroblasts are formed from tissue fibroblasts through the o Foreign bodies (unecessary sutures or fragments
effect of PDGF, TGF-β, and FGF-2, but they can also originate impedes healing)
from bone marrow precursors known as fibrocytes o Size, location and type of wound
 wounds in richly vascularized areas such as the face,
heal faster than those in poorly vascularized ones,
such as the foot
 small incisional injuries heal faster than large
excisional wounds
IV. Pathologic Aspects of Repair
 Complications in wound healing can arise from abnormalities in
any of the basic components of the repair process
 aberrations can be grouped into three general categories: 1)
deficient scar formation, 2) excessive formation of repair
components, and 3) formation of contractures
 Inadequate formation of granulation tissue or assembly of a scar
can lead to wound dehiscence and ulceration
Connective Tissue Remodeling o Dehiscence or rupture of wound is most common after
 Replacement of granulation tissue with a scar involves changes abdominal surgery and is due to increased abdominal
in the composition of the ECM pressure. Vomiting, coughing, or ileus can generate
 The balance between ECM synthesis and degradation results in mechanical stress on the abdominal wound
remodelling of the connective tissue framework - an important o Wounds can ulcerate because of inadequate
feature of tissue repair vascularization during healing

Page 34 of 44
TRANSCRIBERS: Bancod, Catindig, Magaoay, Ordoña
Acute & Chronic Inflammation & Tissue Repair

 Excessive formation of the components of the repair process POST-LECTURE QUIZ


can give rise to hypertrophic scars and keloids
o accumulation of excessive collagen may give rise to a 1. What receptor in the breast is detected in breast cancer?
raised scar known as hypertrophic scar 2. What converts hydrogen peroxide to hypochlorite?
o If scar tissue grows beyond the boundaries of original 3. What molecule is responsible for leukocyte adhesion?
wound and does not regress, it is called a keloid. 4. What molecule is responsible for transmigration or
diapedesis?
o Keloid formation seens to be an individual predisposition,
5. What enzyme is inhibited by NSAIDs?
wherein african americans are more predisposed
6. What is produced upon macrophage activation during acute
o hypertrophic scars generally develop after thermal or phase response?
traumatic injury that involves the dermis 7. What is produced upon macrophage activation during acute
phase response?
8. What disorder is caused by a defect in myeloperoxidase?
9. What disorder is caused by a defect in fibrillar collagen
synthesis?
10. What gene is responsible for regulating apoptosis in the cell
cycle?

Answers:
1.) HER2/ERB2 2.) Myeloperoxidase 3.) ICAM-1 4.) PECAM-1
5.) Cyclooxygenase 6.) TNF 7.) IL-1 8.) Chronic granulomatous
disease 9.) Alport syndrome 10.) p53
 Exuberant granulation is another deviation in wound healing
consisting of the formation of excessive amounts of granulation
tissue, which protrudes above the level of the surrounding skin
and blocks re-epithelialization
 Contraction in the size of a wound is an important part of the
normal healing process. An exaggeration of this process gives
rise to contracture and results in the deformities of the wound
and the surrounding tissues. They are prone to develop in
palms, soles, and the anterior aspect of thorax. Contractures are
commonly seen after serious burns and can compromise joint
movements.
FIBROSIS
 most often indicates the deposition of collagen in chronic
diseases
 The persistence of injury leads to chronic inflammation, which is
associated with the proliferation and activation of macrophages
and lymphocytes, and the production of a plethora of
inflammatory and fibrinogenic growth factors and cytokines.
 TGF-β is practically always involved as an important fibrinogenic
agent.
o produced by most cells in granulation tissue, causes
fibroblast migration and proliferation, increased synthesis
of collagen and fibronectin, and decreased degradation of
ECM due to inhibition of metalloproteinases
o cell death by necrosis or apoptosis and the production of
ROS seem to be important triggers of the activation,
regardless of the tissue
o Recent studies provide evidence for an important role for
osteopontin in wound healing and fibrosis. In animal
experiments, osteopontin blockage decreases formation
of granuloma tissue and scarring.
 Fetal cutaneous wound heal without scar formation
Fibrotic disorders include diverse diseases such as liver cirrhosis,
systemic sclerosis, fibrosing diseases of the lung, chronic
pancreatitis, glomerulonephritis, and constrictive pericarditis.

Page 35 of 44
TRANSCRIBERS: Bancod, Catindig, Magaoay, Ordoña
[EDIT] Lecture Topic 1.02
06/20/2015
ACUTE & CHRONIC INFLAMMATION & TISSUE REPAIR
DR. YABUT
Sample Questions (from Robbins’ Review of Pathology) Answers and Rationale
C. Macrophages in tissues derived from circulating blood monocytes are
1. An 11-year-old child falls and cuts his hand. The wound becomes infected. phagocytic cells that respond to a variety of stimuli, and they represent the
Bacteria extend into the extracellular matrix around capillaries. In the
janitorial crew of the body. The other cells listed are not phagocytes. B cells
inflammatory response to this infection, which of the following cells removes
the bacteria? can differentiate into plasma cells secreting antibodies to neutralize
A. B lymphocyte infectious agents. Fibroblasts form collagen as part of a healing response.
B. Fibroblast Mast cells can release a variety of inflammatory mediators. T cells are a key
C. Macrophage part of chronic inflammatory processes in cell-mediated immune responses.
D. Mast cell
E. T lymphocyte
2. A 53-year-old woman has had a high fever and cough productive of D. These signs and symptoms suggest acute bacterial pneumonia. Such
yellowish sputum for the past 2 days. Her vital signs include temperature of infections induce an acute inflammation dominated by neutrophils that fill
37.8° C, pulse 103/min, respirations 25/min, and blood pressure 100/60 mm alveoli, as shown in the figure, and are coughed up, which gives the sputum
its yellowish, purulent appearance. Langhans giant cells are seen with
Hg. On auscultation of the chest, crackles are audible in both lung bases. A
granulomatous inflammatory responses. Macrophages become more
chest radiograph shows bilateral patchy pulmonary infiltrates. The numerous after initiation of acute events, cleaning up tissue and bacterial
microscopic appearance of her lung is shown in the figure. Which of the debris through phagocytosis. Mast cells are better known as participants in
following inflammatory cell allergic and anaphylactic responses. Lymphocytes are a feature of chronic
types is most likely to be seen inflammation.
in greatly increased numbers
in her sputum specimen?
A. Langhans giant cells
B. Macrophages
C. Mast cells
D. Neutrophils
E. T lymphocytes

E. Nonhuman microbial substances such as double-stranded RNA of


3. A 4-year-old child has had a high-volume diarrhea for the past 2 days. On viruses, bacterial DNA, and bacterial endotoxin, can be recognized by Toll-
examination she is dehydrated. A stool sample examined by serologic assay like receptors (TLRs) on human cells as part of an innate defense
is positive for rotavirus. She is treated with intravenous fluids and recovers.
Which of the following components is found on intestinal cells and rec- mechanism against infection. Caspase-1 is activated by an inflammasome
ognizes double-stranded RNA of this virus to signal transcription factors that complex of proteins responding to bacterial organisms, and produces
upregulate interferon production for viral elimination? biologically active interleukin-1 (IL-1). Complement receptors on
A. Caspase-1 inflammatory cells recognize complement components that aid in triggering
B. Complement receptor immune responses through co-stimulatory signals. Lectins found on cell
C. Lectin surfaces can bind a variety of substances, such as fungal polysaccharides,
D. T cell receptor
that trigger cellular defenses. T cell receptors respond to peptide antigens to
E. Toll-like receptor
trigger a cell-mediated immune response.
E. The formation of an exudate containing a significant amount of protein
4. A 72-year-old man with severe emphysema has had worsening right and cells depends on the ―leakiness‖ of blood vessels, principally venules.
ventricular failure for the past 5 years. For the past 4 days, he has had fever
When exudation has occurred, the protein content of the extravascular space
and increasing dyspnea. A chest radiograph shows an accumulation of fluid
in the pleural spaces. Fluid obtained by thoracentesis has a specific gravity increases, and extravascular colloid osmotic pressure increases, causing
of 1.030 and contains degenerating neutrophils. The most likely cause of this extracellular fluid accumulation. Leukocytosis alone is insufficient for
fluid accumulation is due to changes in which of the following? exudation because the leukocytes must be driven to emigrate from the
A. Colloid osmotic pressure vessels by chemotactic factors. The lymphatics scavenge exuded
B. Leukocytic diapedesis proteinaceous fluid and reduce the amount of extravascular and extracellular
C. Lymphatic pressure fluid. Sodium and water retention helps drive transudation of fluid.
D. Renal sodium retention
E. Vascular permeability
B. Exudation of fluid from venules and capillaries is a key component of the
5. A 35-year-old man has had increasing dyspnea for the past 24 hours. A acute inflammatory process. Several mechanisms of increased vascular
chest radiograph shows large, bilateral pleural effusions. Thoracentesis
permeability have been proposed, including formation of interendothelial
yields 500 mL of slightly cloudy yellow fluid from the right pleural cavity.
Cytologic examination of the fluid shows many neutrophils, but no gaps by contraction of endothelium. This contraction can be caused by
lymphocytes or RBCs. Which of the following mechanisms contributes most mediators such as histamine and leukotrienes. The vessels then become
to the pleural fluid accumulation? more ―leaky,‖ and the fluid leaves the intravascular space to accumulate
A. Arteriolar vasoconstriction extravascularly, forming effusions in body cavities or edema within tissues.
B. Endothelial contraction Arteriolar vasoconstriction is a transient response to injury that helps
C. Inhibition of platelet adherence diminish blood loss. Platelets adhere to damaged endothelium and promote
D. Lymphatic obstruction
hemostasis. Lymphatic obstruction results in the accumulation of protein-rich
E. Neutrophil release of lysosomes
lymph and lymphocytes, producing a chylous effusion within a body cavity.
After neutrophils reach the site of tissue injury outside of the vascular space,
they release lysosomal enzymes that promote liquefaction.

Page 36 of 44
TRANSCRIBERS: Bancod, Catindig, Magaoay, Ordoña
Acute & Chronic Inflammation & Tissue Repair

E. Leukocyte rolling is the first step in transmigration of neutrophils from the


6. A 6-year-old child has a history of recurrent infections with pyogenic vasculature to the tissues. Rolling depends on interaction between selectins
bacteria, including Staphylococcus aureus and Streptococcus pneumoniae.
(P-selectin and E-selectin on endothelial cells, and L-selectin on neutrophils)
The infections are accompanied by a neutrophilic leukocytosis. Microscopic
examination of a biopsy specimen obtained from an area of soft tissue and their sialylated ligands (e.g., sialylated Lewis X). Integrins are involved in
necrosis shows microbial organisms, but very few neutrophils. An analysis of the next step of transmigration, during which there is firm adhesion between
neutrophil function shows a defect in rolling. This child’s increased neutrophils and endothelial cells. Complement C3b acts as an opsonin to
susceptibility to infection is most likely caused by a defect involving which of facilitate phagocytosis. Leukotriene B4 is a chemotactic agent. NADPH
the following molecules? oxidase is involved in phagocytic cell microbicidal activity.
A. Complement C3b
B. Integrins
C. Leukotriene B4
D. NADPH oxidase
E. Selectins
B. Chemokines include many molecules that are chemotactic for neutrophils,
7. In an experiment, bacteria are introduced into a perfused tissue eosinophils, lymphocytes, monocytes, and basophils. Bradykinin causes pain
preparation. Leukocytes leave the vasculature and migrate to the site of
and increased vascular permeability. Complement C3a causes increased
bacterial inoculation. The movement of these leukocytes is most likely to be
mediated by which of the following substances? vascular permeability by releasing histamine from mast cells. Histamine
A. Bradykinin causes vascular leakage. Prostaglandins have multiple actions, but they do
B. Chemokines not cause chemotaxis.
C. Complement C3a
D. Histamine
E. Prostaglandins
B. This immunoglobulin deficiency prevents opsonization and phagocytosis
8. A 12-month-old boy with a 6-month history of repeated infections has had of microbes. Deficiency of integrins and selectins, or a defect in
a fever and cough for the past 3 days. A Gram stain of sputum shows many
microtubules, would prevent adhesion and locomotion of neutrophils. H2O2
gram-positive cocci in chains. CBC shows neutrophilia. Laboratory studies
show that the patient’s neutrophils phagocytose and kill organisms promptly production is part of the oxygen-dependent killing mechanism. This
in the presence of normal human serum, but not in his own serum. The mechanism is intact in this patient because the neutrophils are able to kill
neutrophils migrate normally in a chemotaxis assay. Which of the following is bacteria when immunoglobulins in normal serum allow phagocytosis.
the most likely cause of this boy’s increased susceptibility to infection?
A. Abnormality of selectin expression
B. Diminished opsonization
C. Defective neutrophil generation of hydrogen peroxide
D. Deficiency of integrins
E. Phagocytic cell microtubular protein defect
B. Myeloperoxidase is present in the azurophilic granules of neutrophils. It
9. A 5-year-old child has a history of recurrent bacterial infections, including converts H2O2 into HOCl–, a powerful oxidant and antimicrobial agent.
pneumonia and otitis media. Analysis of leukocytes collected from the Degranulation occurs as phagolysosomes are formed with engulfed bacteria
peripheral blood shows a deficiency in myeloperoxidase. A reduction in in phagocytic vacuoles within the neutrophil cytoplasm. Oxygen consumption
which of the following processes is the most likely cause of this child’s in- with an oxidative or respiratory burst after phagocytosis is aided by glucose
creased susceptibility to infections? oxidation and activation of neutrophil NADPH oxidase, resulting in
A. Hydrogen peroxide (H2O2) elaboration generation of superoxide that is converted by spontaneous dismutation to
B. Hydroxy-halide radical (HOCl–) formation H2O2. In contrast, prostaglandin production depends on a functioning
C. Failure of migration resulting from complement deficiency cyclooxygenase pathway of arachidonic acid metabolism.
D. Phagocytic cell oxygen consumption
E. Prostaglandin production
B. The respiratory, or oxidative, burst of neutrophils generates reactive
10. In an experiment, neutrophils collected from peripheral blood are oxygen species (e.g., superoxide anion) that are important in destruction of
analyzed for a ―burst‖ of oxygen consumption. This respiratory burst is an
engulfed bacteria. This burst can be quantitated by flow cytometric analysis.
essential step for which of the following events in an acute inflammatory
response? Neutrophil attachment to endothelium is aided by adhesion molecules on
A. Attachment to endothelial cells both the endothelium and the neutrophil surface. These molecules include
B. Generation of microbicidal activity selectins and integrins. Myelopoiesis does not depend on generation of
C. Increased production in bone marrow superoxide. Bacteria are opsonized by complement C3b and IgG, allowing
D. Opsonization of bacteria the bacteria to be more readily phagocytosed.
E. Phagocytosis of bacteria
D. During acute inflammation, in the first stage of extravasation, the
11. A 4-year-old girl has had numerous infections with Staphylococcus neutrophils ―roll over‖ the endothelium. At this stage, the adhesion between
aureus since infancy. Genetic testing shows a defect leading to a lack of β2
the neutrophils and endothelial cells is weak. Rolling is mediated by binding
integrin production. Which of the following abnormalities of neutrophil
function is most likely responsible for these clinical symptoms? of selectins to sialylated oligosaccharides. The next step, firm adhesion, is
A. Decreased generation of hydroxy-halide radicals (HOCl–) mediated by binding of integrins on the leukocytes to their receptors,
B. Diminished phagocytosis of bacteria opsonized with IgG intercellular adhesion molecule-1 or vascular cell adhesion molecule-1
C. Failure of migration to the site of infection (VCAM-1), on endothelial cells. Integrins have two chains, α and β. A genetic
D. Inadequate adhesion on cytokine-activated endothelium lack of β chains prevents firm adhesion of leukocytes to endothelial cells.
E. Reduced respiratory burst after phagocytosis This process depends on adhesion molecules expressed on the neutrophils
and endothelial cells. Formation of HOCl– requires myeloperoxidase
released from neutrophil granules. Phagocytosis of opsonized organisms

Page 37 of 44
TRANSCRIBERS: Bancod, Catindig, Magaoay, Ordoña
Acute & Chronic Inflammation & Tissue Repair

depends on engulfment, which requires contractile proteins in the neutrophil


cytoplasm. Neutrophil migration to a site of infection depends on the
presence of chemotactic factors such as complement C5a that bind to the
neutrophil and activate phospholipase C to begin a series of events that
culminate in the influx of calcium, which triggers contractile proteins. The
respiratory burst to kill phagocytized organisms depends on NADPH
oxidase, and a deficiency of this enzyme leads to chronic granulomatous
disease.
E. Monocytes transforming to macrophages contain cytokine-inducible nitric
12. In an experiment, peripheral blood cells are isolated and placed into a oxide synthase (iNOS), which generates nitric oxide. Nitric oxide, by itself
culture medium that preserves their metabolic activity. Interferon-γ is added
and on interaction with other reactive oxygen species, has antimicrobial
to this culture, along with viable Escherichia coli organisms. Which of the
following blood cell types in this medium is the most likely to have activity. CD4 or CD8 lymphocytes can be the source for interferon-γ (IFN-γ),
bactericidal activity against E. coli? which stimulates macrophage production of NOS. Endothelial cells contain a
A. Basophil form of NOS (eNOS) that acts to promote vasodilation. B lymphocytes
B. B lymphocyte produce immunoglobulins that can opsonize bacteria. Basophils release
C. CD4+ lymphocyte histamine and arachidonic acid metabolites, which participate in the acute
D. CD8+ lymphocyte inflammatory process. Natural killer cells have Fc receptors and can lyse
E. Monocyte
IgG-coated target cells; they also generate IFN-γ. Neutrophils can
F. Natural killer cell
G. Neutrophil phagocytize microbes, but they use
B. Interferon-γ secreted from lymphocytes stimulates monocytes and
13. In an experiment, T lymphocytes from peripheral blood are placed in a macrophages, which secrete their own cytokines that further activate
medium that preserves their function. The lymphocytes are activated by
lymphocytes. Interferon-γ also is important in transforming macrophages into
contact with antigen and incubated for 4 hours. The supernatant fluid is
collected and is found to contain a substance that is a major stimulator of epithelioid cells in a granulomatous inflammatory response. Histamine
monocytes and macrophages. Which of the following substances released released from mast cells is a potent vasodilator, increasing vascular
into this fluid medium is most likely to stimulate macrophages? permeability. Leukotriene B4, generated in the lipoxygenase pathway of
A. Histamine arachidonic acid metabolism, is a potent neutrophil chemotactic factor. Nitric
B. Interferon-γ oxide generated by macrophages aids in destruction of microorganisms;
C. Leukotriene B4 nitric oxide released from endothelium mediates vasodilation and inhibits
D. Nitric oxide
platelet activation. Binding of agonists such as epinephrine, collagen, or
E. Phospholipase C
F. Tumor necrosis factor (TNF) thrombin to platelet surface receptors activates phospholipase C, which
catalyzes the release of arachidonic acid from two of the major membrane
phospholipids, phosphatidylinositol and phosphatidylcholine. Tumor necrosis
factor (TNF), produced by activated macrophages, mediates many systemic
effects, including fever, metabolic wasting, and hypotension.
C. Histamine is found in abundance in mast cells, which are normally present
14. A woman who is allergic to cats visits a neighbor who has several cats. in connective tissues next to blood vessels beneath mucosal surfaces in
During the visit, she inhales cat dander, and within minutes, she develops
airways. Binding of an antigen (allergen) to IgE antibodies that have
nasal congestion with abundant nasal secretions. Which of the following
substances is most likely to produce these findings? previously attached to the mast cells by the Fc receptor triggers mast cell
A. Bradykinin degranulation, with release of histamine. This response causes increased
B. Complement C5a vascular permeability and mucous secretions. Bradykinin, generated from
C. Histamine the kinin system on surface contact of Hageman factor with collagen and
D. Interleukin-1 (IL-1) basement membrane from vascular injury, promotes vascular permeability,
E. Phospholipase C smooth muscle contraction, and pain. Complement C5a is a potent
F. Tumor necrosis factor (TNF)
chemotactic factor for neutrophils. Interleukin-1 (IL-1) and tumor necrosis
factor (TNF), both produced by activated macrophages, mediate many
systemic effects, including fever, metabolic wasting, and hypotension.
Phospholipase C, which catalyzes the release of arachidonic acid, is
generated from platelet activation.
F. The COX-2 enzyme is inducible with acute inflammatory reactions,
15. In a 6-month randomized trial of a pharmacologic agent, one group of particularly in neutrophils, in synovium, and in the central nervous system.
patients receives a cyclooxygenase-2 (COX-2) inhibitor, and a control group
The cyclooxygenase pathway of arachidonic acid metabolism generates
does not. Both groups of adult males had mild congestive heart failure and
bilateral symmetric arthritis of small joints. Laboratory measurements during prostaglandins, which mediate pain, fever, and vasodilation. Ankle swelling is
the trial show no significant differences between the groups in WBC count, most likely to result from peripheral edema secondary to congestive heart
platelet count, hemoglobin, and creatinine. The group receiving the drug failure. Increased susceptibility to bruising results from prolonged
reports subjective findings different from those of the control group. Which of glucocorticoid administration, which also causes leukopenia. Asthma results
the following findings was most likely reported by the group receiving the from bronchoconstriction mediated by leukotrienes that are generated by the
drug? lipoxygenase pathway of arachidonic acid metabolism. Inhibition of histamine
A. Increased ankle swelling
released from mast cells helps reduce urticaria. Fever can be mediated by
B. Increased susceptibility to bruising
C. Increased bouts of asthma prostaglandin release, not inhibition.
D. Reduced severity of urticaria
E. Numerous febrile episodes

Page 38 of 44
TRANSCRIBERS: Bancod, Catindig, Magaoay, Ordoña
Acute & Chronic Inflammation & Tissue Repair

F. Reduced arthritis pain


E. Naproxen, a nonsteroidal anti-inflammatory drug, targets the
16. A 19-year-old woman develops a sore throat and fever during the past cyclooxygenase pathway of arachidonic acid metabolism and leads to
day. Physical examination shows pharyngeal erythema and swelling. reduced prostaglandin generation. Prostaglandins promote vasodilation at
Laboratory findings include leukocytosis. She is given naproxen. Which of sites of inflammation. Chemotaxis is a function of various chemokines, and
the following features of the acute inflammatory response is most affected by complement C3b may promote phagocytosis, but neither is affected by
this drug? aspirin. Leukocyte emigration is aided by various adhesion molecules.
A. Chemotaxis Leukocyte release from bone marrow can be driven by the cytokines
B. Emigration interleukin-1 (IL-1) and tumor necrosis factor (TNF).
C. Leukocytosis
D. Phagocytosis
E. Vasodilation
E. Prostaglandins are produced through the cyclooxygenase pathway of
17. A 35-year-old woman takes acetylsalicylic acid (aspirin) for arthritis. arachidonic acid metabolism. Aspirin and other nonsteroidal anti-
Although her joint pain is reduced with this therapy, the inflammatory process
inflammatory drugs block the synthesis of prostaglandins, which can produce
continues. The aspirin therapy alleviates her pain mainly through reduction in
the synthesis of which of the following mediators? pain. Complement C1q is generated in the initial stage of complement
A. Complement C1q activation, which can eventually result in cell lysis. Histamine is mainly a
B. Histamine vasodilator. Leukotrienes are generated by the lipoxygenase pathway, which
C. Leukotriene E4 is not blocked by aspirin. Nitric oxide released from endothelium is a
D. Nitric oxide vasodilator.
E. Prostaglandins
C. Endothelial cells can release nitric oxide to promote vasodilation in areas
18. A 77-year-old woman experiences a sudden loss of consciousness, with of ischemic injury. Bradykinin mainly increases vascular permeability and
loss of movement on the right side of the body. Cerebral angiography shows
produces pain. Leukotriene E4, platelet-activating factor, and thromboxane
an occlusion of the left middle cerebral artery. Elaboration of which of the
following mediators will be most beneficial in preventing further ischemic A2 have vasoconstrictive properties.
injury to her cerebral cortex?
A. Bradykinin
B. Leukotriene E4
C. Nitric oxide
D. Platelet-activating factor
E. Thromboxane A2
A. Activation of complement may occur via microbial cell wall components
19. In an experiment, bacteria are inoculated into aliquots of normal human such as polysaccharides (alternative pathway) or mannose (lectin pathway),
blood that have been treated with an anticoagulant. It is observed that the
or antibody attached to surface antigens (classic pathway). A variety of
bacteria are either phagocytized by neutrophils or undergo lysis. Which of
the following blood plasma components is most likely to facilitate these complement components are generated, including complement C5a, a
effects? neutrophil chemoattractant; complement C3b, an opsonin; and complement
A. Complement C5-9, the membrane attack complex. The remaining options are more
B. Fibrin closely associated with coagulation. Fibrin is generated by the coagulation
C. Kallikrein system, but not with anticoagulation. Kallikrein may aid in generation of
D. Plasmin bradykinin and plasmin, but participates just in complement C5a generation.
E. Thrombin
Plasmin is generated from plasminogen and helps lyse clots. Thrombin is
generated by the coagulation cascade.
E. Ongoing activation of coagulation generates an inflammatory response
20. Patients with extensive endothelial injury from Escherichia coli sepsis that further amplifies coagulation, creating a vicious cycle. Protein C
have consumption of coagulation factors as well as an extensive
antagonizes coagulation factor V, which catalyzes activation of prothrombin
inflammatory response. Administration of activated protein C is most likely to
decrease this inflammatory response by reducing the amount of which of the to thrombin, thereby breaking the cycle of thrombin generation. Complement
following substances? components can become activated by plasmin (C3) and kallikrein (C5),
A. Complement forming anaphylatoxins (C3a and C5a) that promote inflammation. Fibrin, the
B. Fibrin end product of coagulation pathways, forms a meshwork entrapping platelets
C. Kallikrein and creating a plug. Kallikrein is generated by activation of Hageman factor
D. Plasmin (XII) and leads to formation of bradykinin. Plasmin is generated from
E. Thrombin
plasminogen activated by thrombosis to promote clot lysis.
D. Serous inflammation is the mildest form of acute inflammation. A blister is
21. A 95-year-old woman touches a pot of boiling water. Within 2 hours, she a good example of serous inflammation. It is associated primarily with
has marked erythema of the skin of the fingers of her hand, and small
exudation of fluid into the subcorneal or subepidermal space. Because the
blisters appear on the finger pads. This has led to which one of the following
inflammatory responses? injury is mild, the fluid is relatively protein-poor. A protein-rich exudate results
A. Fibrinous inflammation in fibrin accumulation. Granulomatous inflammation is characterized by
B. Granulomatous inflammation collections of transformed macrophages called epithelioid cells. Acute
C. Purulent inflammation inflammatory cells, mainly neutrophils, exuded into a body cavity or space
D. Serous inflammation form a purulent (suppurative) exudate, typically associated with liquefactive
E. Ulceration necrosis. Loss of the epithelium leads to ulceration.
C. This patient is experiencing an acute inflammatory response, with edema,
22. A 24-year-old, sexually active woman has experienced lower abdominal erythema, and pain of short duration. Neutrophils form an exudate and
pain for the past day. Her temperature is 37.9° C, and on palpation, the left
release various proteases, which can produce liquefactive necrosis, starting

Page 39 of 44
TRANSCRIBERS: Bancod, Catindig, Magaoay, Ordoña
Acute & Chronic Inflammation & Tissue Repair

lower abdomen is markedly tender. Laboratory findings include a total WBC at the mucosa and extending through the wall of the tube. This mechanism
count of 29,000/mm3 with 75% segmented neutrophils, 6% bands, 14% results in perforation. Fibroblasts are more likely participants in chronic
lymphocytes, and 5% monocytes. Laparotomy reveals a distended, fluid- inflammatory responses and in healing responses, generally appearing more
filled, reddened left fallopian tube that is about to rupture. A left
than 1 week after the initial event. Langhans giant cells are a feature of
salpingectomy is performed. Which of the following is most likely to be seen
on microscopic examination of the excised fallopian tube? granulomatous inflammation. Mononuclear infiltrates are more typical of
A. Fibroblastic proliferation chronic inflammation of the fallopian tube, in which rupture is less likely.
B. Langhans giant cells Epithelial metaplasia is most likely t
C. Liquefactive necrosis
D. Mononuclear infiltrates
E. Squamous metaplasia
E. Bacterial infections often evoke an acute inflammatory response
23. A 68-year-old man has had worsening shortness of breath for the past dominated by neutrophils. The extravasated neutrophils attempt to
week. On physical examination, his temperature is 38.3° C. On percussion,
phagocytose and kill the bacteria. In the process, some neutrophils die, and
there is dullness over the left lung fields. Thoracentesis performed on the left
pleural cavity yields 800 mL of cloudy yellow fluid that has a WBC count of the release of their lysosomal enzymes can cause liquefactive necrosis of
2500/mm3 with 98% neutrophils and 2% lymphocytes. A Gram stain of the the tissue. This liquefied tissue debris and both live and dead neutrophils
fluid shows gram-positive cocci in clusters. Which of the following terms best comprise pus, or purulent exudate. Such an exudate is typical of bacterial
describes the process occurring in his left pleural cavity? infections that involve body cavities. Another term for purulent exudate in the
A. Abscess pleural space is empyema. An abscess is a localized collection of neutrophils
B. Chronic inflammation within tissues. Chronic inflammation occurs when there is a preponderance
C. Edema
of mononuclear cells, such as lymphocytes, macrophages, and plasma cells,
D. Fibrinous inflammation
E. Purulent exudate in a process that has gone on for more than a few days—more likely weeks
F. Serous effusion or months—or that accompanies repeated bouts of acute inflammation.
Edema refers to increased cellular and interstitial fluid collection within
tissues, leading to tissue swelling. In fibrinous inflammation, exudation of
blood proteins (including fibrinogen, which polymerizes to fibrin) gives a
grossly shaggy appearance to surfaces overlying the inflammation. A serous
effusion is a watery-appearing transudate that resembles an ultrafiltrate of
blood plasma, with a low cell and protein content.
D. If inflammation is limited and brief, and the involved tissue can
24. An 87-year-old woman has had a cough productive of yellowish sputum regenerate, then resolution is the likely outcome, without significant loss of
for the past 2 days. On examination her temperature is 37° C. A chest function. In older persons this may take longer, but can still occur. Multiple
radiograph shows bilateral patchy infiltrates. Her peripheral blood shows bouts of acute inflammation, or ongoing inflammation, can become chronic,
leukocytosis. A week later she is afebrile. Which of the following is the most and there tends to be loss of some tissue function. If significant tissue
likely outcome of her pulmonary disease? destruction occurs, there is likely to be formation of a fibrous scar in the
A. Chronic inflammation region of the tissue loss. Acute inflammation is not a preneoplastic event.
B. Fibrous scarring Ulceration refers to loss of an epithelial surface with acute inflammation; if
C. Neoplasia the epithelium regenerates, then there is resolution.
D. Resolution
E. Ulceration
F. Inflammation involving an epithelial surface may cause such extensive
25. A 53-year-old woman has experienced abdominal pain for 2 weeks. She necrosis that the surface becomes eroded, forming an ulcer. If the
is afebrile. There is mild upper abdominal tenderness on palpation, and inflammation continues, the ulcer can continue to penetrate downward into
bowel sounds are present. An upper gastrointestinal endoscopy is submucosa and muscularis. Alternatively, the ulcer may heal, or it may
performed. The figure shows remain chronically inflamed. An abscess is a localized collection of
microscopic examination of a neutrophils in tissues. A caseating granuloma is granulomatous inflammation
biopsy specimen of a duodenal with central necrosis; the necrosis has elements of both liquefaction and
lesion. Which of the following coagulative necrosis. Chronic inflammation occurs when there is a
pathologic processes is most preponderance of mononuclear cells, such as lymphocytes, macrophages,
likely present? and plasma cells, in a process that has gone on for more than a few days—
A. Abscess more likely weeks or months—or that accompanies repeated bouts of acute
B. Caseating granuloma inflammation. Pus, or a purulent exudate, appears semiliquid and yellowish
C. Chronic inflammation because of the large numbers of granulocytes present. A serous effusion is a
D. Purulent exudate watery-appearing transudate that resembles an ultrafiltrate of blood plasma,
E. Serous effusion with a low cell and protein content.
F. Ulceration
A. The rounded density in the right lower lobe of the lung has liquefied
26. A 92-year-old woman is diagnosed with Staphylococcus aureus contents that form a central air-fluid level. There are surrounding infiltrates.
pneumonia and receives a course of antibiotic therapy. Two weeks later, she
The formation of a fluid-filled cavity after infection with Staphylococcus
no longer has a productive cough, but she still has a temperature of 38.1° C.
A chest radiograph shows the findings in the figure. Which of the following aureus suggests that liquefactive necrosis has occurred. The cavity is filled
terms best describes the outcome of the patient’s pneumonia? with tissue debris and viable and dead neutrophils (pus). Localized, pus-filled
A. Abscess formation cavities are called abscesses. Some bacterial organisms, such as S. aureus,
B. Complete resolution are more likely to be pyogenic, or pus-forming. With complete resolution, the
C. Fibrous scarring structure of the lung remains almost unaltered. Scarring or fibrosis may
D. Chronic inflammation follow acute inflammation as the damaged tissue is replaced by fibrous
E. Tissue regeneration
connective tissue. Most bacterial pneumonias resolve, and progression to

Page 40 of 44
TRANSCRIBERS: Bancod, Catindig, Magaoay, Ordoña
Acute & Chronic Inflammation & Tissue Repair

continued chronic inflammation is uncommon. Lung tissue, in contrast to


liver, is incapable of regeneration, except for epithelium and endothelium.
E. This patient has an infective endocarditis with septic embolization,
27. A 29-year-old woman with a congenital ventricular septal defect has had producing a cerebral abscess. The tissue destruction that accompanies
a persistent temperature of 38.6° C and headache for the past 3 weeks. A
abscess formation as part of acute inflammatory processes occurs from
head CT scan shows an enhancing 3-cm, ring like lesion in the right parietal
lobe of her brain. Which of the following actions by inflammatory cells has lysosomal enzymatic destruction, aided by release of reactive oxygen
most likely produced this CT finding? species. Nitric oxide generated by macrophages aids in destruction of infec-
A. Elaboration of nitric oxide by macrophages tious agents. Immunoglobulin formed by B cells neutralizes and opsonizes
B. Formation of immunoglobulin by B lymphocytes infectious agents. Prostaglandins produced by endothelium promote
C. Generation of prostaglandin by endothelium vasodilation. Interferon-γ released from lymphocytes plays a major role in
D. Production of interferon-γ by T lymphocytes chronic and granulomatous inflammatory responses.
E. Release of lysosomal enzymes from neutrophils
B. One outcome of acute inflammation with ulceration is chronic
28. A 37-year-old man has had midepigastric pain for the past 3 months. An inflammation. This is particularly true when the inflammatory process
upper gastrointestinal endoscopy shows a 2-cm, sharply demarcated,
continues for weeks to months. Chronic inflammation is characterized by
shallow ulceration of the gastric antrum. Microscopic examination of a biopsy
from the ulcer base shows angiogenesis, fibrosis, and mononuclear cell in- tissue destruction, mononuclear cell infiltration, and repair. In acute
filtrates with lymphocytes, macrophages, and plasma cells. Which of the inflammation, the healing process of fibrosis and angiogenesis has not
following terms best describes this pathologic process? begun. In fibrinous inflammation, typically involving a mesothelial surface,
A. Acute inflammation there is an outpouring of protein-rich fluid that results in precipitation of fibrin.
B. Chronic inflammation Granulomatous inflammation is a form of chronic inflammation in which
C. Fibrinous inflammation epithelioid macrophages form aggregates. Serous inflammation is an
D. Granulomatous inflammation
inflammatory process involving a mesothelial surface (e.g., lining of the
E. Serous inflammation
pericardial cavity), with an outpouring of fluid having little protein or cellular
content.
C. Macrophages, present in such lesions, play a prominent role in the
29. A 65-year-old man develops worsening congestive heart failure 2 weeks healing process. Activated macrophages can secrete various cytokines that
after an acute myocardial infarction. An echocardiogram shows a markedly
promote angiogenesis and fibrosis, including platelet-derived growth factor,
decreased ejection fraction. Now, capillaries, fibroblasts, collagen, and
inflammatory cells have largely replaced the infarcted myocardium. Which of fibroblast growth factor, interleukin-1 (IL-1), and tumor necrosis factor (TNF).
the following inflammatory cell types in this lesion plays the most important Eosinophils are most prominent in allergic inflammations and in parasitic
role in the healing process? infections. Epithelioid cells, which are aggregations of activated
A. Eosinophils macrophages, are typically seen with granulomatous inflammation, and the
B. Epithelioid cells healing of acute inflammatory processes does not involve granulomatous in-
C. Macrophages flammation. Neutrophils are most numerous within the initial 48 hours after
D. Neutrophils
infarction, but are not numerous after the first week. Plasma cells can
E. Plasma cells
secrete immunoglobulins and are not instrumental to healing of an area of
tissue injury.
B. Interferon-γ is secreted by activated T cells and is an important mediator
30. A 9-year-old boy has had a chronic cough and fever for the past month. of granulomatous inflammation. It causes activation of macrophages and
A chest radiograph shows enlargement of hilar lymph nodes and bilateral
their transformation into epithelioid cells and then giant cells. Complement
pulmonary nodular interstitial infiltrates. A sputum sample contains acid-fast
bacilli. A transbronchial biopsy specimen shows granulomatous inflammation C3b acts as an opsonin in acute inflammatory reactions. Interleukin-1 (IL-1)
with epithelioid macrophages and Langhans giant cells. Which of the can be secreted by macrophages to produce various effects, including fever,
following mediators is most likely to contribute to giant cell formation? leukocyte adherence, fibroblast proliferation, and cytokine secretion.
A. Complement C3b Leukotriene B4 induces chemotaxis in acute inflammatory processes. Tumor
B. Interferon-γ necrosis factor (TNF) can be secreted by activated macrophages and
C. Interleukin-1 (IL-1) induces activation of lymphocytes and proliferation of fibroblasts, which are
D. Leukotriene B4
other elements of a granuloma.
E. Tumor necrosis factor (TNF)
D. These findings suggest a granulomatous inflammation, and tuberculosis is
31. A 32-year-old woman has had a chronic cough with fever for the past a common cause. Candida is often a commensal organism in the
month. On physical examination, her temperature is 37.5° C. A chest
oropharyngeal region and rarely causes pneumonia in healthy (non-
radiograph shows many small, ill-defined nodular opacities in all lung fields.
A transbronchial biopsy specimen shows interstitial infiltrates with immunosuppressed) individuals. Viral infections tend to produce a
lymphocytes, plasma cells, and epithelioid macrophages. Which of the mononuclear interstitial inflammatory cell response. Bacteria such as
following infectious agents is the most likely cause of this appearance? Enterobacter and Staphylococcus are more likely to produce acute
A. Candida albicans inflammation. Plasmodium produces malaria, a parasitic infection without a
B. Cytomegalovirus significant degree of lung involvement.
C. Enterobacter aerogenes
D. Mycobacterium tuberculosis
E. Plasmodium falciparum
F. Staphylococcus aureus
D. The polarizable material is the suture, and a multinucleated giant cell
32. One month after an appendectomy, a 25-year-old woman palpates a reaction, typically with foreign body giant cells, is characteristic of a
small nodule beneath the skin at the site of the healed right lower quadrant
granulomatous reaction to foreign material. Granulation tissue may form a
sutured incision. The nodule is excised, and microscopic examination shows
macrophages, collagen deposition, small lymphocytes, and multinucleated nodular appearance, and begins to appear 3 to 5 days following injury, but is

Page 41 of 44
TRANSCRIBERS: Bancod, Catindig, Magaoay, Ordoña
Acute & Chronic Inflammation & Tissue Repair

giant cells. Polarizable, refractile material is seen in the nodule. Which of the unlikely to persist for a month. Chronic inflammation alone is unlikely to
following complications of the surgery best accounts for these findings? produce a localized nodule with giant cells. Edema refers to accumulation of
A. Abscess formation fluid in the interstitial space. It does not produce a cellular nodule. If a large,
B. Chronic inflammation
gaping wound is not closed by sutures, it can granulate it and myofibroblastic
C. Exuberant granulation tissue
D. Granuloma formation contraction eventually helps close the wound by second intention.
E. Healing by second intention
C. Figure A shows diffuse reticulonodular pulmonary densities, and Figure B
33. A 43-year-old man has had a cough and fever for the past 2 months. A shows noncaseating granulomas with many epithelioid cells and two
chest CT scan shows the findings in the figure (A). A transbronchial lung
prominent large Langhans giant cells. If special stains and/or cultures for
biopsy is performed, yielding a specimen with the microscopic appearance
shown in the figure (B). Which of organisms (usually mycobacteria or fungi) are negative, then this is likely
the following chemical mediators sarcoidosis. Macrophage stimulation and transformation to epithelioid cells
is most important in the and giant cells are characteristic of granuloma formation. Interferon-γ
pathogenesis of this lesion? promotes the formation of epithelioid cells and giant cells. Bradykinin is
A. Bradykinin released in acute inflammatory responses and results in pain. Complement
B. Complement C5a C5a is chemotactic for neutrophils. Although occasional neutrophils are seen
C. Interferon-γ
in granulomas, neutrophils do not form a major component of granulomatous
D. Nitric oxide
E. Prostaglandins inflammation. Macrophages can release nitric oxide to destroy other cells,
but nitric oxide does not stimulate macrophages to form a granulomatous re-
sponse. Prostaglandins are mainly involved in the causation of vasodilation
and pain in acute inflammatory responses.
E. The findings here are those of strep throat with acute inflammation.
34. An 8-year-old girl has had difficulty swallowing for the past day. On Bacterial organisms often lead to fever accompanying infection through
examination, her pharynx is swollen and erythematous with an overlying
release of exogenous pyrogens that induce inflammatory cells to release
yellow exudate. Laboratory studies show neutrophilia. Streptococcus
pyogenes (group A streptococcus) is cultured from her pharynx. Which of the endogenous pyrogens such as tumor necrosis factor (TNF) and interleukin-1
following substances is most likely to increase in response to pyrogens (IL-1). The pyrogens stimulate prostaglandin synthesis in the hypothalamus
released by this organism? to ―reset the thermostat,‖ so that fever occurs as a sign of the acute
A. Hageman factor inflammatory response. Hageman factor initiates the coagulation cascade.
B. Immunoglobulin E Immunoglobulin E is often increased in response to inflammatory responses
C. Interleukin-12 (IL-12) with allergens and with invasive parasites. Interleukin-12 (IL-12) released by
D. Nitric oxide
macrophages stimulates T-cell responses. Nitric oxide generated in
E. Prostaglandins
endothelium leads to vasodilation, whereas nitric oxide produced in
macrophages aids in microbial killing.
E. Fever is produced by various inflammatory mediators, but the major
35. A 41-year-old man has had a severe headache for the past 2 days. On cytokines that produce fever are interleukin-1 (IL-1) and tumor necrosis
examination, his temperature is 39.2° C. A lumbar puncture is performed,
factor (TNF), which are produced by macrophages and other cell types. IL-1
and the cerebrospinal fluid obtained has a WBC count of 910/mm3 with 94%
neutrophils and 6% lymphocytes. Which of the following substances is the and TNF can have autocrine, paracrine, and endocrine effects. They mediate
most likely mediator for the fever observed in this man? the acute phase responses, such as fever, nausea, and neutrophil release
A. Bradykinin from bone marrow. Bradykinin, generated from the kinin system on surface
B. Histamine contact of Hageman factor with collagen and basement membrane from
C. Leukotriene B4 vascular injury, promotes vascular permeability, smooth muscle contraction,
D. Nitric oxide and pain. Histamine released from mast cells is a potent vasodilator,
E. Tumor necrosis factor (TNF)
increasing vascular permeability. Leukotriene B4, generated in the
lipoxygenase pathway of arachidonic acid metabolism, is a potent neutrophil
chemotactic factor. Nitric oxide generated by macrophages aids in
destruction of microorganisms; nitric oxide released from endothelium
mediates vasodilation and inhibits platelet activation.
B. This acute inflammatory process leads to production of acute-phase
36. A 43-year-old man with a ventricular septal defect has had a cough and reactants, such as C-reactive protein (CRP), fibrinogen, and serum amyloid
fever for the past 2 days. On examination, he has a temperature of 37.6° C
A (SAA) protein. These proteins, particularly fibrinogen, and
and a cardiac murmur. A blood culture grows Streptococcus, viridans group.
His erythrocyte sedimentation rate (ESR) is increased. Microbial cells are immunoglobulins increase RBC rouleaux formation to increase the
opsonized and cleared. Which of the following chemical mediators is most erythrocyte sedimentation rate (ESR), which is a nonspecific indicator of
important in producing these findings? inflammation. CRP production is upregulated by interleukin-6 (IL-6), whereas
A. Bradykinin fibrinogen and SAA are upregulated mainly by tumor necrosis factor (TNF)
B. C-reactive protein and interleukin-1 (IL-1). Interferon-γ is a potent stimulator of macrophages.
C. Interferon-γ Nitric oxide can induce vasodilation or can assist in microbial killing within
D. Nitric oxide
macrophages. Prostaglandins are vasodilators.
E. Prostaglandin
F. Tumor necrosis factor (TNF)
C. Hepatocytes are stable cells with an extensive ability to regenerate. The
37. In an experiment, a group of test animals is infected with viral hepatitis. ability to restore normal architecture of an organ such as the liver depends
Two months later, complete recovery of the normal liver architecture is
on the viability of the supporting connective tissue framework. If the connec-
observed microscopically. A control test group is infected with bacterial
organisms, and after the same period of time, fibrous scars from resolving tive tissue cells are not injured, hepatocyte regeneration can restore normal

Page 42 of 44
TRANSCRIBERS: Bancod, Catindig, Magaoay, Ordoña
Acute & Chronic Inflammation & Tissue Repair

hepatic abscesses are seen microscopically. Which of the following factors liver architecture. This regeneration occurs in many cases of viral hepatitis. A
best explains the different outcomes for the two test groups? liver abscess associated with liquefactive necrosis of hepatocytes and the
A. Extent of damage to the biliary ducts supporting connective tissue heals by scarring. The other options listed may
B. Extent of the hepatocyte injury
explain the amount of liver injury, but not the nature of the response.
C. Injury to the connective tissue framework
D. Location of the lesion within the liver
E. Nature of the injurious etiologic agent
A. Hepatocytes are quiescent (stable) cells that can reenter the cell cycle
38. A 51-year-old woman tests positive for hepatitis A antibody. Her serum and proliferate in response to hepatic injury, enabling the liver to regenerate
AST level is 275 U/L, and ALT is 310 U/L. One month later, these enzyme
partially. Acute hepatitis results in hepatocyte necrosis, marked by elevations
levels have returned to normal. Which phase of the cell cycle best describes
the hepatocytes 1 month after her infection? in AST and ALT. After the acute process has ended, cells return to the G0
A. G0 phase, and the liver becomes quiescent again.
B. G1
C. S
D. G2
E. M
A. Vitamin C deficiency leads to scurvy, with reduced lysyl oxidase enzyme
39. A 54-year-old man undergoes laparoscopic hernia repair. In spite of the activity that helps cross-link fibrillar collagens to provide tensile strength.
small size of the incisions, he has poor wound healing. Further history
Though elastin is a fibrillar protein, it tends to regenerate poorly in scar
reveals that his usual diet has poor nutritional value and is deficient in
vitamin C. Synthesis of which of the following extracellular matrix tissue, even with the best of nutrition, explaining why a scar does not stretch
components is most affected by this deficiency? like the skin around it. The other listed choices are glycoproteins that have
A. Collagen an adhesive quality and are not vitamin C dependent.
B. Elastin
C. Fibronectin
D. Integrin
E. Laminin
F. The figure shows a subacute infarction with granulation tissue formation
40. In an experiment, glass beads are embolized into the coronary arteries of containing numerous capillaries stimulated by vascular endothelial growth
rats, resulting in myocardial injury. After 7 days, sections of the myocardium factor, representing a healing response. Epidermal growth factor aids in
are studied using light microscopy. The microscopic appearance of one of reepithelialization of a surface wound. Interleukin-2 (IL-2) mediates
these sections is shown in
lymphocyte activation. Leukotriene B4 mediates vasoconstriction and
the figure. Which of the
following mediators is most bronchoconstriction. Thromboxane A2 aids vasoconstriction and platelet
likely being expressed to aggregation. Tumor necrosis factor (TNF) induces endothelial activation and
produce this appearance? many responses that occur secondary to inflammation, including fever, loss
A. Epidermal growth factor of appetite, sleep disturbances, hypotension, and increased corticosteroid
B. Interleukin-2 (IL-2) production.
C. Leukotriene B4
D. Thromboxane A2
E. Tumor necrosis factor
(TNF)
F. Vascular endothelial
growth factor
A. At 1 week, wound healing is incomplete, and granulation tissue is still
41. A 20-year-old woman undergoes cesarean section to deliver a term present. More collagen is synthesized in the following weeks. Wound
infant, and the lower abdominal incision is sutured. The sutures are removed
strength peaks at about 80% by 3 months. Type IV collagen is found in
1 week later. Which of the
following statements best basement membranes.
describes the wound site at the
time of suture removal?
A. Collagen degradation exceeds
synthesis
B. Granulation tissue is still
present
C. No more wound strength will
be gained
D. Type IV collagen
predominates
E. Wound strength is 80% of normal tissue

E. The figure shows dense collagen with some remaining dilated blood
42. A 24-year-old man with acute appendicitis undergoes surgical removal of vessels, typical of the final phase of wound healing, which is extensive by the
the inflamed appendix. The incision site is sutured. A trichrome-stained end of the first month. On day 1, the wound is filled only with fibrin and
section representative of the site with blue appearing collagen is shown in inflammatory cells. Macrophages and granulation tissue are seen 2 to 3 days
the figure. How long after the surgery would this appearance most likely be postoperatively. Neovascularization is most prominent by days 4 and 5. By
seen? week 2, collagen is prominent, and fewer vessels and inflammatory cells are
A. 1 day
seen.
B. 2 to 3 days

Page 43 of 44
TRANSCRIBERS: Bancod, Catindig, Magaoay, Ordoña
Acute & Chronic Inflammation & Tissue Repair

C. 4 to 5 days
D. 2 weeks
E. 1 month
D. Wound contraction is a characteristic feature of healing by second
43. A 40-year-old man underwent laparotomy for a perforated sigmoid colon intention that occurs in larger wounds. Collagen synthesis helps fill the
diverticulum. A wound infection complicated the postoperative course, and
defect, but does not contract it. Adhesive glycoproteins such as fibronectin
surgical wound dehiscence occurred. Primary closure was no longer pos-
sible, and the wound ―granulated in.‖ Six weeks later, the wound is only 10% help to maintain a cellular scaffolding for growth and repair, but they do not
of its original size. Which of the following processes best accounts for the contract. The inhibition of metalloproteinases leads to decreased
observed decrease in wound size over the past 6 weeks? degradation of collagen and impaired connective tissue remodeling in wound
A. Elaboration of adhesive glycoproteins repair. Edema diminishes over time, but this does not result in much
B. Increase in synthesis of collagen contraction.
C. Inhibition of metalloproteinases
D. Myofibroblast contraction
E. Resolution of subcutaneous edema
C. Integrins interact with the extracellular matrix proteins (e.g., fibronectin).
44. In an experiment involving observations on wound healing, researchers Engagement of integrins by extracellular matrix proteins leads to the
noted that intracytoplasmic cytoskeletal elements, including actin, interact
formation of focal adhesions where integrins link to intracellular cytoskeletal
with the extracellular matrix to promote cell attachment and migration in
wound healing. Which of the following substances is most likely responsible elements such as actin. These interactions lead to intracellular signals that
for such interaction between the cytoskeleton and the extracellular matrix? modulate cell growth, differentiation, and migration during wound healing.
A. Epidermal growth factor Epidermal growth factor stimulates epithelial cell and fibroblast proliferation.
B. Fibronectin Platelet-derived growth factor (PDGF) can be produced by endothelium,
C. Integrin macrophages, smooth muscle cells, and platelets; PDGF mediates migration
D. Platelet-derived growth factor and proliferation of fibroblasts and smooth muscle cells and migration of
E. Type IV collagen
monocytes. Type IV collagen is found in basement membranes on which
F. Vascular endothelial growth factor
cells are anchored. Vascular endothelial growth factor promotes
angiogenesis (capillary proliferation) through endothelial cell proliferation and
migration in a healing response.
A. Glucocorticoids inhibit wound healing by impairing collagen synthesis.
45. A 23-year-old woman receiving corticosteroid therapy for an autoimmune This is a desirable side effect if the amount of scarring is to be reduced, but it
disease has an abscess on her upper outer right arm. She undergoes minor
results in the delayed healing of surgical wounds. Angiogenesis driven by
surgery to incise and drain the abscess, but the wound heals poorly over the
next month. Which of the following aspects of wound healing is most likely to vascular endothelial growth factor (VEGF) is not significantly affected by
be deficient in this patient? corticosteroids. Neutrophil infiltration is not prevented by glucocorticoids.
A. Collagen deposition Reepithelialization, in part driven by epidermal growth factor, is not affected
B. Elaboration of VEGF by corticosteroid therapy. Serine proteinases are important in wound
C. Neutrophil infiltration remodeling.
D. Reepithelialization
E. Serine proteinase production
B. The healing process sometimes results in an exuberant production of
46. An 18-year-old man lacerated his left collagen, giving rise to a keloid, which is a prominent raised, nodular scar, as
ear and required sutures. The sutures shown in the figure. This tendency may run in families. Dehiscence occurs
were removed 1 week later. Wound when a wound pulls apart. Organization occurs as granulation tissue is
healing continued, but the site became
replaced by fibrous tissue. If normal tissue architecture is restored, resolution
disfigured over the next 2 months by the
process shown in the figure. Which of of inflammation has occurred. Secondary union describes the process by
the following terms best describes the which large wounds fill in and contract.
process that occurred in this man?
A. Dehiscence
B. Keloid formation
C. Organization
D. Resolution
E. Secondary union

D. The elevated creatine kinase level indicates that myocardial necrosis has
47. A 58-year-old man had chest pain persisting for 4 hours. A radiographic occurred. A fibrous scar gradually replaces the area of myocardial necrosis.
imaging procedure showed an infarction involving a 4-cm area of the
Chronic inflammation is typically driven by ongoing stimuli such as persistent
posterior left ventricular free wall. Laboratory findings showed serum creatine
kinase of 600 U/L. Which of the following pathologic findings would most infection, autoimmunity, or irritation from endogenous or exogenous
likely be seen in the left ventricular lesion 1 month later? chemical agents, and it is not a feature of ischemic myocardial injury.
A. Chronic inflammation Coagulative necrosis is typical of myocardial infarction, but after 1 month, a
B. Coagulative necrosis scar would be present. The destruction of myocardial fibers precludes
C. Complete resolution complete resolution. Nodular regeneration is typical of hepatocyte injury
D. Fibrous scar because hepatocytes are stable cells.
E. Nodular regeneration

Page 44 of 44
TRANSCRIBERS: Bancod, Catindig, Magaoay, Ordoña

Vous aimerez peut-être aussi